Evidence Flashcards

You may prefer our related Brainscape-certified flashcards:
1
Q

The prosecution cannot initiate evidence of the
defendant’s bad character. The prosecution may offer such evidence only…

A

…after the accused has put his character in issue by either taking the stand (thus placing his credibility in issue) or offering
evidence of his good character. Thus

How well did you know this?
1
Not at all
2
3
4
5
Perfectly
2
Q

The driver’s witness is prepared to testify that the driver has a
reputation for being a safe and prudent driver, which the driver plans to use to prove that, because he has a propensity for driving safely, he was in fact driving safely at the time of the accident.

Permissible?

A

This is a classic case of the impermissible use of propensity evidence.

The general rule is that
evidence of character traits (here, safety and prudence) is inadmissible in a civil case

to prove that a party acted in conformity with those traits on a particular occasion. [Fed. R. Evid. 404(a)]
This case fits squarely within that general rule.

How well did you know this?
1
Not at all
2
3
4
5
Perfectly
3
Q

Evidence of character to prove the conduct of a person in the litigated event is generally not admissible in a civil case. Here, the plaintiff is trying to employ the circumstantial use of prior behavior patterns to draw the inference that the defendant drove at an excessive rate of speed at the time of the incident here at issue. Such a use of character evidence is not permitted.

Exception?

A

An exception to the general prohibition
of character evidence is that, when proof of a person’s character, as a matter of substantive law, is an essential element of a claim or defense in the case, character evidence is admissible.

The defendant’s character as a driver is not in issue; rather, his actions at a specific time and place are in issue.

i.e.

The principal issue is whether D as driving too fast for the wet conditions.

How well did you know this?
1
Not at all
2
3
4
5
Perfectly
4
Q

In a criminal case, other crimes and wrongs of the defendant may be admissible even though
they are not charged, but they are not automatically admissible. There are two basic principles:

A
  1. Other crimes or wrongs are not admissible to show that the defendant is a bad person, nor are they admissible to show propensity of the defendant to commit this crime.
  2. Other crimes or wrongs may be admissible if they are relevant to show motive, intent, absence of mistake, identity or common
    scheme or plan (MIMIC), _unless the judge determines that the probative value is substantially outweighed
    by prejudice.
    _
How well did you know this?
1
Not at all
2
3
4
5
Perfectly
5
Q

In a civil case, evidence of character to prove the conduct of a person in the litigated event is generally not admissible. The slight probative value of character is outweighed by the dangers of prejudice and distracting the jury from the main issues. Therefore, circumstantial use of prior behavior patterns for the purpose of drawing the inference that a person has a particular character trait and that, at the time and place in question, she probably acted in conformity with it is not permitted.

Example?

A

Evidence of the defendant’s good driving record is being offered to show that she is a careful driver and to raise the inference that, when the accident occurred, she was acting in conformity with that trait.
This constitutes impermissible use of character evidence and is inadmissible.

How well did you know this?
1
Not at all
2
3
4
5
Perfectly
6
Q

A witness may be impeached
with evidence of a prior conviction for …

A

(i) any felony or (ii) any crime involving dishonesty or false statement.

Because the conviction was for a crime of dishonesty (fraudulent business practices),
and the conviction is less than 10 years old, the question is proper on cross-examination because
it goes to the defendant’s credibility.

How well did you know this?
1
Not at all
2
3
4
5
Perfectly
7
Q

When does the 10-year time limit for previous convictions not apply?

A

Because the defendant did not take the stand, this evidence is not being offered for impeachment and, thus, the 10-year
time limit does not apply.

Evidence of other crimes is admissible against an accused in a criminal
case if it is relevant to some issue other than the defendant’s character or disposition to commit the
crime charged. Where, as here, the crime charged is embezzlement, evidence that the defendant
committed embezzlement before might be admissible to establish fraudulent intent.

How well did you know this?
1
Not at all
2
3
4
5
Perfectly
8
Q

When analyzing questions involving the admissibility of
hearsay evidence, ask two things:

A

(i) Is the proffered evidence hearsay (i.e., an out-of-court statement by a declarant, being offered to prove the truth of the matter asserted in the statement)?
(ii) If hearsay, is the proffered evidence nevertheless admissible because it fits within an exception to the hearsay rule?
i. e.

The witness’s proffered testimony is hearsay because it matters whether the defendant’s statement is true or false. If his statement is true (if the defendant actually intended to go to the state capital, as stated), then the likelihood that he actually went there is increased.
(This is because people tend to act in a manner consistent with their previously stated intentions.)
Since the witness’s testimony would be used for the purpose of establishing the truth of the defendant’s statement, it is hearsay evidence. However, the testimony fits within a hearsay
exception. A declarant’s statement of present intention to take an action in the future fits within the “state of mind” exception contained in Federal Rule of Evidence 803(3).

How well did you know this?
1
Not at all
2
3
4
5
Perfectly
9
Q

Information in treatises can be read into evidence if the treatise is:

A

(i) relied upon by the expert or
is called to his attention during cross-examination; and

(ii) established as reliable by the witness,
another expert, or judicial notice.

How well did you know this?
1
Not at all
2
3
4
5
Perfectly
10
Q

statement of P’s then-existing medical condition

v.

statement made for purposes of medical diagnosis and treatment

A

There are two hearsay exceptions involving statements of physical condition:

(i) statements of the declarant’s present physical condition, regardless of the purpose for which the statement is
made; and

(ii) statements made for the purpose of obtaining medical diagnosis or treatment.

a. Declarant’s Present Bodily Condition
A spontaneous declaration of a declarant’s own present bodily condition is admissible as an exception to the hearsay rule regardless of whether it was made for the purpose of
diagnosis or treatment.

b. Statements Made for Diagnosis or Treatment—Includes Past Conditions
A statement that describes a person’s medical history, past or present symptoms, or their inception or general cause is admissible as an exception to the hearsay rule if
it was made for—and was reasonably pertinent to—medical diagnosis or treatment.
Usually a declarant will be describing her own condition, but this is not required (e.g., the declarant may be seeking medical assistance for a family member).

How well did you know this?
1
Not at all
2
3
4
5
Perfectly
11
Q

Witness gave description of D to police officer. D charged w/ criminal counts and acquitted.

P sued in civil court, but Witness died, so P wanted to introduce into evidence police report containing the description.

Admissible?

A

The court should not admit the report because it is hearsay not within any exception. The report contains an out-of-court statement being offered for its truth; i.e., that the person who hit the victim fits the description given by the witness.

The report does not fall within any exception to the hearsay rule. It is not a business record because the witness was not
under a business duty to convey the information to the police.

How well did you know this?
1
Not at all
2
3
4
5
Perfectly
12
Q

Copies of judgments …. hearsay

A

are

(because they are out-of-court statements used to prove the truth of the matter asserted).

But, the Federal Rules of Evidence provide that a judgment of a felony conviction is admissible as an exception to the hearsay rule in both criminal and civil actions to prove
any fact essential to the judgment. The Rules define felony convictions as crimes punishable by death or imprisonment in excess of one year. [Fed. R. Evid. 803(22)]

The copy of the judgment is not deemed to be a public record
for purposes of the hearsay exception for public records and reports because

the copy of the judgment of conviction is not a record, report, statement, or data compilation of a public office
or agency, setting forth:

(i) the activities of the office or agency,
(ii) matters observed pursuant to a duty imposed by law, or (
iii) factual findings resulting from an investigation made pursuant to authority granted by law.

How well did you know this?
1
Not at all
2
3
4
5
Perfectly
13
Q

Where an out-of-court statement is introduced for any
purpose other than to prove the truth of the matter asserted, the statement is not hearsay. One
type of out-of-court statement that is not hearsay is evidence of legally operative facts.

Examples?

A

These are utterances to which legal significance is attached, such as words of contract, bribery, or
cancellation.

How well did you know this?
1
Not at all
2
3
4
5
Perfectly
14
Q

Dead Man Acts provide that…

A

…a person interested in an event is incompetent to testify to a personal transaction or communication with a deceased, when such testimony is offered against the representative or successors in interest of the decedent.

How well did you know this?
1
Not at all
2
3
4
5
Perfectly
15
Q

Impeachment by prior inconsistent statement - when won’t it work?

A

When the witness has simply testified that he does not remember either seeing the defendant take the jewelry or telling the plaintiff that she did so.

Impeachment refers to the casting of an adverse reflection on the truthfulness of a witness. One form of impeachment is to
show that a witness has, on another occasion, made statements that are inconsistent with some material part of his present testimony.

If the witness in his testimony had denied seeing the defendant
take anything or telling the plaintiff that she had done so, then the testimony of the plaintiff as to the witness’s previous statements would be admissible as a prior inconsistent statement, thus serving to disprove the credibility of the witness. However, because the witness has merely testified
to a lack of memory concerning these matters, the plaintiff’s testimony probably would not
be considered a prior inconsistent statement.

How well did you know this?
1
Not at all
2
3
4
5
Perfectly
16
Q

A specific act of misconduct offered to
attack the witness’s character for truthfulness can be elicited only on…

A

…cross-examination.

Applies to a witness AND D, who takes a stand.

If the witness denies the act, the cross-examiner cannot refute the answer by calling other witnesses or producing other evidence.

How well did you know this?
1
Not at all
2
3
4
5
Perfectly
17
Q

Bias or adverse interest can be proved by…

A

…cross-examination or extrinsic evidence, and in some cases, both.

How well did you know this?
1
Not at all
2
3
4
5
Perfectly
18
Q

May any witness be impeached by

(i) any felony conviction (unless the judge determines that its probative value is substantially outweighed by Rule 403 considerations) or

(ii) conviction of any other crime requiring proof or
admission of an act of dishonesty or false statements?

A

Only witness other than the accused.

In a criminal case in which the accused is being impeached, a felony conviction that does not involve dishonesty or a
false statement will be admitted only if the government shows that its probative value as impeachment
evidence outweighs its prejudicial effect.

How well did you know this?
1
Not at all
2
3
4
5
Perfectly
19
Q

Federal Rule of Evidence 608(b) permits cross-examination as to prior bad acts if, in the discretion
of the trial court,…

A

…they are probative of truthfulness.

How well did you know this?
1
Not at all
2
3
4
5
Perfectly
20
Q

Prior inconsistent statements, made under oath and subject to the penalty of perjury in a deposition or prior hearing, are admissible as …

A

…nonhearsay.

How well did you know this?
1
Not at all
2
3
4
5
Perfectly
21
Q

The credibility of a hearsay declarant may be attacked by evidence that would be admissible if the declarant had …

A

…testified as a witness.

i.e.

The pedestrian’s (hearsay declarant’s) statement to the motorist may be impeached by proof that he made the inconsistent statement to the medical technician.

How well did you know this?
1
Not at all
2
3
4
5
Perfectly
22
Q

A witness’s character for truthfulness
may be attacked by a conviction for any felony, even if …

A

….the felony did not involve dishonesty or false statement. Because this was a felony conviction and is less than 10 years old, it is likely to be allowed on cross-examination.

Note, however, that the trial court retains discretion under Rule
403 to exclude a felony conviction for a crime that does not involve dishonesty or false statement
if its probative value as impeachment evidence is substantially outweighed by the danger of unfair prejudice.

How well did you know this?
1
Not at all
2
3
4
5
Perfectly
23
Q

Witness consulted her diary the night before her testimony. Should the testimony be stricken?

A

Under the Federal Rules,
any materials can be used to refresh one’s recollection, and the Rules do not prohibit the use of such materials before trial.

How well did you know this?
1
Not at all
2
3
4
5
Perfectly
24
Q

A question is leading and generally objectionable when it suggests to the witness the fact that the
examiner expects and wants to have confirmed.

When are leading questions not permitted?

A

Leading questions are generally not permitted on
direct examination of a disinterested witness.

i.e.

Asking the eyewitness on direct as to whether the light was red is objectionable as leading.

How well did you know this?
1
Not at all
2
3
4
5
Perfectly
25
Q

An unresponsive answer by a witness is subject to a motion to …. by ….

A

… strike by examining counsel.

Thus, examining counsel can adopt an unresponsive answer if it is not objectionable on
some other ground.

How well did you know this?
1
Not at all
2
3
4
5
Perfectly
26
Q

When a court calls a witness, each party is entitled to…

A

…..cross-examine the witness.

How well did you know this?
1
Not at all
2
3
4
5
Perfectly
27
Q

May a court call a witness on its own initiative?

A

A court may call a witness at a party’s request or on its own initiative.

How well did you know this?
1
Not at all
2
3
4
5
Perfectly
28
Q

the basis of an expert’s opinion …. be admissible into evidence for the expert’s testimony to be admissible.

A

need not

provided that the information is of a type reasonably relied upon by experts in the particular field.

How well did you know this?
1
Not at all
2
3
4
5
Perfectly
29
Q

A layperson’s opinion is admissible if…

A

(1) it is rationally based on the perception of the witness,
(2) helpful to a clear understanding of the witness’s testimony on the determination of a fact in issue, and

(3) not based on scientific, technical, or other specialized knowledge.
(1) not satisfied when witness is speculating rather than testifying to his own perceptions.

How well did you know this?
1
Not at all
2
3
4
5
Perfectly
30
Q

Present recollection refreshed

v.

Past recollecton recorded

A

Under the rule of present recollection refreshed,

a witness may be shown any writing or other thing that may refresh her memory of an event. The writing is not authenticated, is not in evidence, and may be used solely to refresh her recollection.

The doctrine of past recollection recorded applies when

a party is seeking to introduce a memorandum or other record into evidence. To have the record read into
evidence, a foundation must be laid.

Ask: Is the party seeking to introduce the document into evidence?

How well did you know this?
1
Not at all
2
3
4
5
Perfectly
31
Q

Attorney has been carrying on such cross-examination for three days, with the prospect of at least one more day. Can the trial court terminate it?

A

The trial court has the authority to cut off cross-examination when it determines there has been an adequate opportunity
for meaningful cross-examination.

How well did you know this?
1
Not at all
2
3
4
5
Perfectly
32
Q

The rancher tells his son in the letters that the rancher and his wife are committing several violations of the law and subjecting themselves to civil liability if they were to get caught.

What exception?

A

Statements made against interest. Basically, if a declarant has made a statement that is so contrary to his interests that the truth of the statement is not in question, the statement is admissible hearsay. It only applies if the declarant is unavailable.

How well did you know this?
1
Not at all
2
3
4
5
Perfectly
33
Q

Is defendant’s bar receipt hearsay?

A

Hearsay is any out of court statement offered in court to prove the truth of the matter asserted. The statement can either be oral, written or conduct intended to be a speech substitute. Here, the receipt is an out of court statement. If it is being offered to prove that the defendant was drinking at the bar, it is hearsay. Thus, if offered to prove she was intoxicated the receipt becomes hearsay and is inadmissible.

How well did you know this?
1
Not at all
2
3
4
5
Perfectly
34
Q

present sense impressions

v.

recorded recollection

A

A statement qualifies under the exception if it is a statement that explains or describes an event or condition made while or immediately after it was perceived.

i.e. The woman wrote down her observation of the smell, heat and smoke as she observed it.

Recorded recollections are an exception to the hearsay rule if it is a record that: (1) the witness once knew about but cannot recall at trial; (2) was made by the witness when the event was fresh in the witness’s mind; and (3) accurately reflects the witness’s knowledge.

i.e. Here, the facts state that the witness is prepared to testify about her abduction and her notes. There is no indication that she does not remember the fire.

How well did you know this?
1
Not at all
2
3
4
5
Perfectly
35
Q

May treatises be used as substantive evidence?

A

FRE 803(18) provides that statements contained in treatises may be admitted into evidence during direct or cross-examination of an expert witness if: (1) the treatise is established as a reliable authority; and (2) the treatise is called to the attention of the expert witness during cross-examination or is relied upon by the expert in direct testimony.

As an exception to the hearsay rule, statements in treatises admitted pursuant to FRE 803(18) may be used as substantive evidence.

How well did you know this?
1
Not at all
2
3
4
5
Perfectly
36
Q

Under FRE 412(b)(1), the defense may submit evidence of a victim’s sexual behavior for the purpose of showing that someone other than the defendant was the source of the victim’s injuries, but, in order to do so,

A

the defense must file a motion describing the evidence 14 days before trial (or at another time set by the court).

How well did you know this?
1
Not at all
2
3
4
5
Perfectly
37
Q

Best evidence rule

A

Under Rule 1002 of the Federal Rules of Evidence, in order to prove the contents of a writing, recording, or photograph, the original writing, recording, or photograph is required, subject to numerous exceptions. This best evidence rule (so named because the original is supposedly the best evidence of the contents) does not apply merely because the writing records or otherwise reflects the happening of an event. Instead, the rule applies only when the writing, recording, or photo is the controlling instrument (e.g., deed, will, or contract) or when the witness is testifying about facts he or she read (e.g., a letter, a license plate number, etc.). In addition, the rule does not apply to collateral matters; as such, when the contents of a writing, recording, or photograph are not closely related to a controlling issue, the contents may be proved by other than production of the original. By definition, the best evidence rule would not apply to a physical object such as the genie lamp.

How well did you know this?
1
Not at all
2
3
4
5
Perfectly
38
Q

The general rule is that character evidence is inadmissible. However, there are some important exceptions, including the exception in criminal trials when the defendant “opens the door.”

A

If the defendant in a criminal trial chooses to admit evidence of his pertinent trait, then the prosecutor can offer evidence to rebut it and character evidence becomes admissible for that purpose. The method of proving character evidence can be in the form of reputation or opinion. It can only be in the form of specific instances if a person’s character trait is an essential element of a charge or defense.

How well did you know this?
1
Not at all
2
3
4
5
Perfectly
39
Q

Are Prior inconsistent statements admissible as substantive evidence?

A

Prior inconsistent statements are admissible as substantive evidence (and not excluded as hearsay) if

the declarant testifies at the trial and is subject to cross-examination, and the prior statement was given under oath at a prior proceeding.

If they do not meet these standards, prior inconsistent statements are hearsay that is admissible for the limited purpose of impeachment. \

How well did you know this?
1
Not at all
2
3
4
5
Perfectly
40
Q

Abuse of discretion.

v.

De novo.

v.

Clearly erroneous.

A

A trial court’s ruling on discretionary matters, such as the admissibility of evidence, is subject to review under the abuse of discretion standard.

Generally, the de novo standard of review is used on appeal for legal rulings. This means that the appellate court reviews the evidence and law without deference to the trial court’s legal rulings.

Generally, the clearly erroneous standard of review is used on appeal for findings of facts. This means that the appellate court must give deference to the trial court’s factual findings.

How well did you know this?
1
Not at all
2
3
4
5
Perfectly
41
Q

Habit evidence

Mnemonic

A

SRA

Specific situation

Repeated

semi-Automatic Response to…

Evidence of a person’s habit may be admitted to prove that on a
particular occasion the person acted in accordance with that habit.

i.e.

According to the testimony, the defendant regularly fails to obey the stop sign at the intersection at which the collision occurred, and in fact, he regularly disregards any stop sign.

How well did you know this?
1
Not at all
2
3
4
5
Perfectly
42
Q

A 60-year-old employee who was fired by a
corporation after 25 years of employment filed
an age discrimination suit against the corporation.
While the corporation’s excuse was that the
reorganization and merger required a trimming
of personnel, at trial the employee seeks to have
a board member testify that the chairman of
the board had convinced the board to fire the
employee because he “didn’t fit our corporate
image of youthful vigor.” It was typical practice
that all directors’ meetings be recorded, and that
the corporate secretary use the recording to type
up a formal transcript of the proceedings. The
meeting at which the employee’s dismissal was
discussed was no exception.
If the defense objects to the board member’s
proposed testimony, how should the court rule?

ANALYSIS!!!!

A

The board member’s testimony should be admissible because it is relevant nonhearsay based on firsthand knowledge.

  1. The testimony is relevant to the employee’s age discrimination suit because it is being offered to prove that the board’s motivation in firing the employee was his age.
  2. It is not hearsay, even though the board member is repeating the statement of an out-of-court declarant (the chairman of the board), because it is not being offered to prove the truth of what the chairman was asserting (i.e., that the employee was in fact too old to fit the corporate image). Rather, it is being offered to show its effect on the board; i.e., it is being offered as circumstantial evidence of the board’s motivation in deciding to fire the employee, which is the critical issue in the case.
  3. Finally, the board member is competent to testify to the chairman’s statement since he heard it firsthand, and no other restrictions on the admissibility of relevant evidence are applicable in this case.
How well did you know this?
1
Not at all
2
3
4
5
Perfectly
43
Q

Admissibility of photographs

A

Photographs are admissible only if identified by a witness as a portrayal of certain facts relevant to the issue and verified by the witness as a correct representation of those facts.

The witness who identifies the photograph need only be familiar with the scene or object that is depicted. It is not necessary to call the photographer to authenticate the photograph.

How well did you know this?
1
Not at all
2
3
4
5
Perfectly
44
Q

Is report of an accident prepared by client at his L’s request privileged?

A

A business report prepared as a communication from client to attorney is privileged.

How well did you know this?
1
Not at all
2
3
4
5
Perfectly
45
Q

A singer has denied his purported signature
on a letter that has become critical in a breach of
contract suit between him and a record producer.
At trial, the record producer’s counsel calls a
teacher who testifies that she taught the singer
mathematics in school 10 years earlier, knows
his signature, and proposes to testify that the
signature to the letter is that of the singer. The
singer’s counsel objects.
The trial judge should:

A

The judge should overrule the objection because lay opinion is permissible (and often essential) to identify handwriting. A foundation must first be laid to establish familiarity with the
handwriting

The fact that the teacher has not seen the singer’s handwriting for 10 years goes to the
credibility of her testimony but not to its admissibility.

How well did you know this?
1
Not at all
2
3
4
5
Perfectly
46
Q

A plaintiff applied for a life insurance policy
and was required to submit to a physical examination
to qualify for the policy. During the
course of the examination, the plaintiff told the
physician, who was approved by the life insurance
company and had never seen the plaintiff
before, “I used to have some back trouble, but
that’s all cleared up now.” A few weeks after the
examination, the defendant’s automobile struck
the rear end of a car in which the plaintiff was
riding as a passenger. The plaintiff now claims
that he suffers persistent lower back pain and
sues the defendant for damages. After laying a
proper foundation that the plaintiff is attempting
to perpetrate a fraud, the defendant calls the
physician as a witness and seeks to have her
testify as to the plaintiff’s statement to her. The
plaintiff’s attorney objects on the ground of the
jurisdiction’s physician-patient privilege.
Should the court allow the physician to testify
about the plaintiff’s statement?

A

The physician’s testimony regarding the plaintiff’s statement is admissible because an examination for insurance purposes is not considered to be for diagnosis and treatment.

To be privileged, the information must be acquired by the physician in the course of treatment.

Thus, the plaintiff’s statement to the physician is not privileged. Since it is not privileged and qualifies as an admission by a party-opponent for hearsay purposes, the
statement is admissible.

How well did you know this?
1
Not at all
2
3
4
5
Perfectly
47
Q

If a physician examines a client at the request of the attorney (e.g., to assess the extent of injury), does the attorney-client
privilege apply?

A

If a physician examines a client at the request of the attorney (e.g., to assess the extent of injury), the attorney-client privilege applies to communications made to the physician because the physician is deemed to be a representative of the attorney.

How well did you know this?
1
Not at all
2
3
4
5
Perfectly
48
Q

A plaintiff sued a defendant for injuries
arising out of a collision between vehicles driven
by the parties. The plaintiff alleged that the
defendant ran a red light when he struck the
plaintiff’s vehicle in an intersection. The plaintiff
wishes to call a witness to the stand who was
near the intersection at the time of the accident.
The witness is prepared to testify that the defendant
offered to pay the witness $500 to testify
falsely in the defendant’s favor.
Is the testimony admissible?

A

Testimony regarding the defendant’s attempt to bribe the witness is admissible as substantive evidence against the defendant. Under the Federal Rules, a statement made by a party and offered against that party (commonly called an admission) is not hearsay. Various kinds of conduct,
including attempts to bribe witnesses, may be held to manifest an awareness of liability or guilt.

Because the defendant’s liability is the issue (i.e., a relevant fact) in the case, his attempt to bribe the witness is admissible as a statement of a party-opponent.

How well did you know this?
1
Not at all
2
3
4
5
Perfectly
49
Q

A state legislator was the chairman of a
committee that disbursed funds to schools in the
state for various projects. The federal government
supplied a portion of the funds as part of a
federal revenue sharing plan. The legislator was
charged with a violation of federal law when
he and his committee made a $10,000 grant for
textbooks to a private school for whites only.
The legislator’s defense is that as chairman of
this committee he was acting in the course of his
legislative duties, and thus, immune from federal
interference.
What is the best argument that would support
the legislator’s constitutional claim?

A

(C) The Tenth Amendment prevents the federal
government from interfering with a member
of the state’s legislature in the performance
of his legislative duties.

How well did you know this?
1
Not at all
2
3
4
5
Perfectly
50
Q

A plaintiff sued his neighbor over a 10-foothigh
stockade fence that the neighbor was
building adjacent to the plaintiff’s backyard.
The local zoning ordinance permitted a fence of
this height unless it was a “spite fence,” defined
as a fence erected solely for the purpose of
interfering with neighboring landowners’ use
and enjoyment of their property. The plaintiff
alleged that the neighbor was building the fence
to block sunlight to the garden that the plaintiff
had planted. The neighbor denied that she was
building the fence for that purpose. The plaintiff
wishes to introduce evidence that the neighbor
had sprayed herbicide towards the garden previously.
Should the judge permit the plaintiff’s testimony?

(B) Yes, because it pertains to the neighbor’s
motivation in building the fence.
(C) No, because the plaintiff’s testimony is
evidence of specific conduct, which is not
admissible in this case because the neighbor’s
character is not in issue.

A

(B) The judge should permit the plaintiff’s testimony because evidence of specific acts of misconduct is admissible to show motive. Under Federal Rule 404(b), evidence of other acts may be admissible in a criminal or civil case if they are relevant to some issue other than character, such as motive. Here, whether the neighbor was motivated by an improper purpose in building the fence is the key issue in the lawsuit by the plaintiff. The neighbor’s prior misconduct in spraying herbicide toward the plaintiff’s garden is circumstantial evidence that her hostility toward the garden motivated her to build the fence.

(C) and (D) are wrong even though they correctly state general rules: evidence of specific acts of misconduct is generally inadmissible, and character evidence is generally inadmissible
in a civil case. However, when the specific acts are being offered for a purpose other than to
show bad character or conduct in conformity to character
, they are admissible in both criminal
and civil cases.

How well did you know this?
1
Not at all
2
3
4
5
Perfectly
51
Q

While a driver was driving someone else’s car,
he hit a plaintiff who was walking in a pedestrian
right-of-way. The plaintiff sued both the
driver and the owner of the car, alleging that the
driver had negligently driven the car and that the
owner had negligently permitted an unfit driver
to use her car. At trial, the plaintiff calls his first
witness. The witness testifies that within the last
several months he is aware of three instances in
which the driver has engaged in reckless driving.
Both the driver and the owner object to the
admission of this evidence.
How should the court rule on the objection?
(A) Sustained, because the driver’s character is
not in issue.

(C) Overruled as to the case against the owner,
but sustained as to the case against the
driver.

A

(C) The witness’s testimony of three instances of reckless driving by the driver would be considered character evidence. Character evidence is not admissible in a civil case if offered to show that a party probably acted in conformity with that character. Character evidence is admissible in a civil case when the character of a person is an issue in the case. The plaintiff is suing the owner on a negligent entrustment theory, and thus the driver’s character as a safe driver is in issue in the case against the owner, but not in the case against the driver himself.

_(_A) is wrong; as stated, the driver’s character is in issue in determining whether the owner was negligent.

52
Q

In a civil action against a security guard and
the bank that employed him, the plaintiff alleged
that he was shot after a traffic altercation when
the security guard jumped out of his car and was
waving his loaded gun. The plaintiff alleged that
the bank was negligent in entrusting the weapon
to the guard, and that the guard was negligent in
his handling of the weapon. The plaintiff offers
the testimony of the guard’s former co-worker,
who worked with the guard for 10 years at
another bank. The former co-worker is prepared
to testify that, during the time that he worked
with the guard, the guard had a reputation for
being a hothead, keeping his weapon loaded
during off-duty hours, and threatening people
with his gun whenever he got into an argument.
Assuming proper objection, how should the
court rule regarding the admissibility of the
testimony?

(B) The former co-worker’s testimony is
character evidence admissible against the
bank if it can be established that the bank
knew of the guard’s reputation.
(C) The former co-worker’s testimony is
character evidence admissible against the
bank whether or not the bank knew of the
guard’s reputation.

A

(C) The former co-worker’s testimony is admissible character evidence. Under Federal Rule 404, in a civil case, evidence of the character of a person generally is inadmissible if offered to prove that the person may have acted in conformity with his character on a particular occasion. If, however, the character evidence is offered for some other purpose, such as where a person’s character itself is an essential element of a claim or defense in the case (e.g., in defamation or negligent hiring/ entrustment cases), Rule 404 will not exclude the evidence. The testimony is evidence of the guard’s character but, if offered against the bank, it would be offered to show that the bank may have been negligent when it entrusted the gun to the guard. Thus, the evidence would not be excluded by Rule 404.

In addition, the evidence would be relevant even if the bank did not know of the guard’s reputation, because the jury could find that a reasonable investigation by the bank would have uncovered the information and the bank should have known of the guard’s reputation.

53
Q

In separate lawsuits, the driver of a car and the
passenger both sued the defendant for injuries
sustained when the defendant’s car struck the
driver’s car. In the passenger’s lawsuit, he testified that he was not paying attention to the other cars on the road at the time of the accident, but offers into evidence the prior deposition testimony of the driver. At that deposition, the driver had testified that she noticed the defendant’s car swerving in the lane next to her and suddenly crossing into her lane and colliding with her car. The driver had moved out of state shortly after her lawsuit concluded. No attempt was made to locate the driver or procure her attendance. Should the court admit the deposition testimony over the defendant’s objection?
(A) Yes, because the statement was made under
oath at a prior deposition.
(B) Yes, under the former testimony exception
to the hearsay rule.
(C) No, because it is hearsay not within any
exception.

A

(C) The court should not admit the deposition testimony. The driver’s statement is hearsay because it is being offered to prove the truth of the matter asserted—that the defendant’s car crossed lanes and struck her car. Because it is hearsay, the statement is admissible only if it falls within an exception to the hearsay rule. Here, there are no exceptions under which to admit the deposition transcript. (A) and (B) are incorrect because the former testimony exception to the hearsay rule does not apply. This exception allows the prior statement of a declarant to be admitted into evidence if the statement was made under oath at a prior deposition or proceeding. There must be a sufficient similarity of parties and issues. The exception applies, however, only if the declarant is unavailable to testify. For the absent driver to be “unavailable” here as defined by the Federal Rules, the passenger must show that he has been unable to procure her attendance or testimony by process or other reasonable means, and here no attempt was made to procure her attendance.
As such, the deposition transcript is inadmissible under this exception and the driver should
be subpoenaed to testify on behalf of the plaintiff.

54
Q

A defendant is charged with the burglary of
a warehouse. At the request of the police investigating the burglary, the night watchman at the warehouse who had seen the thief leaving the premises wrote out a description of the thief,
who bore a strong likeness to the defendant.
However, the night watchman died of a heart
attack before the defendant was arrested and
brought to trial. The prosecution attempts to
offer the description written out by the night
watchman into evidence. Is the description admissible?

(B) Yes, as an identification of a person the
night watchman knew committed the crime
in question.
(C) No, because it is hearsay not within an
exception.

A

(C) The description is inadmissible as hearsay not within an exception.

(B) is wrong. Under the Federal Rules of Evidence, a prior identification is considered nonhearsay, but only if the declarant testifies at the trial and is subject to cross-examination. Since the night watchman died, this requirement cannot be satisfied. On these facts, there are no exceptions to the hearsay rule that would make the description admissible.

55
Q

For the purpose of impeaching the credibility of a witness, a party may show that the witness has, on
another occasion, made statements that are inconsistent with some material part of his present
testimony. In most cases, prior inconsistent statements are hearsay, admissible only to impeach the witness. Exception?

A

However, under the Federal Rules, if the prior inconsistent statement was made under penalty of perjury at a prior trial, hearing, or deposition, it is not hearsay.

56
Q

If statement is not being offered for its truth, but rather to show its effect on the
defendant, can it be offered to show provocation?

A

Yes, i.e. to show provocation for the battery.

57
Q

A man sued a nightclub and one of its
bouncers for injuries he suffered in a physical
altercation with the bouncer as he tried to enter
the nightclub without paying the required cover
charge. At trial, the man’s attorney seeks to
call a police officer to testify that the nightclub’s
manager, who witnessed the altercation,
made the following statement to the officer the
morning after the altercation: “The bouncers are
trained to restrain any troublemakers until the
police arrive, but this time it went too far.” Both
defendants object on the basis of hearsay.
Is the officer’s testimony admissible?
(A) Yes, against the bouncer only.
(B) Yes, against both defendants.
(C) Yes, against the nightclub only.
(D) No, against either defendant.

A

(C) The officer’s testimony is admissible against the nightclub only. A statement by or attributable to an opposing party is not hearsay. A statement by an employee concerning any matter within the
scope of her employment, made during the existence of the employment relationship, is admissible against the employer as a vicarious opposing party’s statement. Here, the manager was an employee of the nightclub when she made the statement about the bouncer’s training and conduct, and such matters were within the scope of her employment. The statement does fit the general definition of hearsay because it was made out of court and is being offered for its truth—that the bouncer went too far during the altercation. However, an opposing party’s statement is categorized as nonhearsay. Consequently, the manager’s statement is admissible against the nightclub. However, statements of a party are not receivable against its co-plaintiffs or co-defendants merely
because they happen to be joined as parties to the action. If there are two or more parties, the
statement of one is receivable against it but, in the absence of authority, not against its co-party.
Thus, the manager’s statement is not admissible against the bouncer.

58
Q

A homeowner returned home from work one
day to find a robber in her living room. After a
brief physical altercation, the homeowner ran
to a bedroom, hid in a closet, and called 911 on
her cell phone. Police officers arrived in less
than two minutes and were able to apprehend
the robber as he tried to run out the front door.
Once they made sure he was locked in the police
car, one of the officers went to speak with the
homeowner about what had happened. She was
still crying and shaking when the officer found
her, and she said, “Thank you for catching him!
He punched me in the head as I was running
away!” The robber was charged with robbery
and assault. Traumatized, the homeowner left the
country and cannot be traced, despite the efforts
of the prosecutor. The prosecutor intends to call
the officer to testify as to the homeowner’s statement. Should the court allow the officer’s testimony?
(A) Yes, because the homeowner’s statement is
an excited utterance.

(C) No, because admitting the homeowner’s
statement would violate the defendant’s
constitutional rights.

A

(C) The testimony should not be admitted.

Under the Confrontation Clause, an accused has the right to be confronted by the witnesses against him. A hearsay statement will not be admitted—even if it falls within a hearsay exception—when: (i) the statement is offered against the accused in a criminal case; (ii) the declarant is unavailable; (iii) the statement was testimonial in nature; and (iv) the accused had no opportunity to cross-examine the declarant’s “testimonial” statement prior to trial.

The Supreme Court has established that if the primary purpose of police interrogation is to enable the police to help in an ongoing emergency, statements made in the course of the
interrogation are nontestimonial. When the primary purpose of the interrogation is to establish or prove past events potentially relevant to a later criminal prosecution, statements are testimonial.
Here, the homeowner’s statement was hearsay because it was made out of court and is being
offered for its truth—that the robber punched her in the head. It appears that the emergency had
already resolved by the time the statement was made. Although only a few minutes had passed
since the physical altercation and the homeowner was still upset, the robber no longer posed any danger because he was locked in the police car and the homeowner was aware of this (“Thank you for catching him!”). Therefore, the homeowner’s statements to the officer were testimonial. Because the homeowner is unavailable to testify at trial and the robber has had no opportunity to cross-examine the statements, admitting them at trial through the testimony of the officer would violate the Confrontation Clause.

(C) The statement likely qualifies as an excited utterance. However, even a hearsay statement that falls within an exception may be barred by the Confrontation Clause, as is the case here.

59
Q

In a medical malpractice action, the plaintiff
sought to have her neighbor testify that, the day
after receiving treatment on her back from the
defendant, the plaintiff told the neighbor that her
back was getting worse.
Upon proper objection, how should the court
rule on this testimony?

(B) Admit it, because it is a statement of a thenexisting
physical condition.
(C) Exclude it, because it is hearsay not within
any exception.

A

(B) The testimony will most likely be admissible as a statement of a then-existing physical condition.

Declarations of existing physical condition are admissible as an exception to the hearsay rule.
Under the Federal Rules, these declarations are admissible even though not made in the context
of medical diagnosis or treatment.

(C) is incorrect. The statement would be hearsay but it would qualify under the exception for statements describing current physical condition.

60
Q

The defendant is being tried for murder in the
bludgeoning death of his brother. The defendant
denies any involvement in the crime. He calls
a witness to the stand, who testifies that, in his
opinion, the defendant is a nonviolent, peaceable
man. Which of the following, if offered by the
prosecution, would most likely be admissible?

(B) A police officer’s testimony that the defendant
has a general reputation in the community as a violent person.

(D) Evidence that the defendant

A

(B) Testimony of the defendant’s reputation as a violent person is admissible to rebut the defendant’s character evidence.

The general rule is that the prosecution cannot initiate evidence of the bad character of the defendant merely to show that he is more likely to have committed the crime of which he is accused. However, if the defendant puts his character in issue by having a character witness testify as to his opinion of the defendant, the prosecution may rebut with evidence of the
defendant’s bad character. One means of rebutting a defendant’s character evidence is by calling qualified witnesses to testify to the defendant’s bad reputation for the particular trait involved in the case.

Here, the defendant put his character in issue by having his witness testify to the
defendant’s nonviolent nature, which is relevant to whether he committed the crime charged. The
prosecution, assuming that it can show that the police officer has knowledge of the defendant’s
reputation in the community, can have the officer testify that the defendant had a reputation as a
violent person.

(D) is incorrect because the basic rule is that when a person is charged with one crime, extrinsic evidence of his other crimes or misconduct is inadmissible if offered solely to establish a criminal disposition, regardless of whether the defendant has placed his character in issue. [Fed. R. Evid.
404(b)] While evidence of other crimes is admissible if it is independently relevant to some other
issue (e.g., motive, intent, or identity), the defendant’s battery conviction in this case appears to
have no relevance other than as evidence of his violent disposition. It is therefore inadmissible.

61
Q

A defendant is on trial for stealing jewelry
from his co-worker. The defendant claims that
the co-worker sold the jewelry to him because
she needed money to buy medicine for her sick
mother. The defense witness is asked to testify as
to the co-worker’s reputation in the community.
The witness testifies that the co-worker is known
as a dishonest person who makes her living as a
“con artist.”
Assuming appropriate objections by defense
counsel, which of the following questions would
NOT be proper on cross-examination of the
witness by the prosecutor?

(B) “Isn’t it true that you were charged last year
with assault for striking your wife?”
(C) “Have you heard that the defendant’s
co-worker teaches Sunday School classes
on morality and has received an award from
her church based on her outstanding moral
character?”
(D) “Do you know that the defendant’s
co-worker teaches Sunday School classes
on morality and has received an award from
her church based on her outstanding moral
character?”

A

(B) Asking the witness about the assault charge is an improper method of impeachment.

A witness may be interrogated upon cross-examination with respect to an act of misconduct only if it is probative of truthfulness. An assault is not probative of truthfulness, so it would not be proper impeachment evidence. Had the witness been convicted of the assault, the conviction would have been admissible, provided it was a felony.

(C) and (D) are incorrect because these questions represent proper means of rebutting the evidence of the co-worker’s character
for dishonesty, as well as trying to impeach the witness’s credibility based on lack of knowledge.
Once the defendant has introduced evidence of the alleged victim’s bad character for apertinent trait, the prosecution may counter with reputation or opinion evidence of the victim’s
good character for the same trait. [Fed. R. Evid. 404] On cross-examination, the prosecution may
inquire into relevant specific instances of conduct. [Fed. R. Evid. 405(a)] Traditionally, asking
a witness if he has heard of a particular instance of conduct represents a means of testing the
accuracy of the hearing and reporting of a reputation witness, who relates what he has heard.
Asking a witness if he knows of a particular instance of conduct is a means of testing the basis
of an opinion expressed by the witness. Here, the witness’s testimony indicates both that he has
heard that the co-worker has a bad reputation for honesty and that his own opinion is that she
is a dishonest person. Thus, in attempting to rebut this testimony, the prosecution may test the
accuracy of what the witness has heard concerning the co-worker’s character by asking him if he
has heard of specific instances of her teaching Sunday School and receiving a church award. Also,
the prosecution may test the basis for the witness’s opinion as to the co-worker’s dishonesty by
asking if he knows of these specific instances that are indicative of her good character.

62
Q

While cross-examining a defendant on trial
for robbery and assault with a deadly weapon,
the prosecutor asks him whether he was
convicted of fraud within the previous year.
Is this question proper?

(A) No, because fraud is not probative of a
tendency to commit violence.

(D) Yes, because it tends to show that the defendant would lie.

A

(D) The question is proper.

The defendant has taken the stand in his own defense, and therefore the prosecutor can attack his credibility as a witness. Under Federal Rule 609, evidence of conviction of a crime requiring proof of an act of dishonesty or false statement can always be used to attack a witness’s character for truthfulness.

(A) is incorrect because even if fraud were probative of the tendency to commit violence, evidence of other crimes is not admissible to prove that a person has a propensity to commit criminal acts.

63
Q

A plaintiff sued a defendant for defamation,
asserting in her complaint that the defendant had
called the plaintiff a thief in front of a number
of business associates. The plaintiff calls two
witnesses to the stand, both of whom testify that
they heard the defendant refer to the plaintiff as
a thief in front of the business associates. The
plaintiff does not take the stand herself. The
defendant pleads truth of the statement as an
affirmative defense and calls a witness to the
stand. The defense witness is prepared to testify
that he was a co-worker of the plaintiff when the
plaintiff supplemented her income by tending
bar three nights a week. The witness will testify
that he saw the plaintiff take a $20 bill from the
tavern’s cash register and secrete the money in
her pocket. The plaintiff’s attorney objects.
May the defense witness’s testimony be
allowed?
(A) Yes, as substantive evidence that the plaintiff
is, in fact, a thief.
(B) Yes, because theft is a crime indicating
dishonesty.

A

(A) The defense witness’s testimony is admissible character evidence because the plaintiff’s character is directly in issue in the case.

As a general rule, evidence of character to prove the conduct of a person in the litigated event is not admissible in a civil case. However, when proof of a person’s character, as a matter of substantive law, is an essential element of a claim or defense in a civil action, character evidence is admissible because it is the best method of proving the issue. Under the Federal Rules, any of the types of evidence—reputation, opinion, or specific acts—may be used.

Here, character is an issue in the plaintiff’s defamation action because the defendant has
pleaded as an affirmative defense that his statement claiming that the plaintiff is a thief is the truth. The defense witness’s testimony that he saw the plaintiff take the money from the cash
register is relevant because it tends to show that the defendant spoke the truth. Hence, it should
be allowed.

(B) is incorrect because the fact that the theft here could be considered a crime of dishonesty would be relevant only if the plaintiff’s credibility were being impeached, and only then if proof of an actual conviction were provided. Here, the testimony is admissible because it is being offered as substantive evidence of an aspect of the plaintiff’s character that is an essential element of a defense in the case.

64
Q

The plaintiff sued a local restaurant, claiming
that she injured her teeth, gums, and mouth
when she bit into a hamburger that contained a
large, jagged piece of glass. The plaintiff called
to the stand a waiter for the restaurant, who
testified that, when he heard the plaintiff scream,
he looked in her direction and saw her remove
a piece of glass from her bleeding mouth. On
cross-examination, the defense asked the waiter,
“Isn’t it a fact that three months ago you were
fired by the restaurant for serving drinks to
your friends and not charging for them?” The
waiter responded, “Yes, but I wasn’t trying to
steal anything. I just forgot to charge them.” The
defense then asked, “Isn’t it a fact that last month
you threw a rock through the plate glass window
at the restaurant?” The waiter replied, “That’s
not true; I was there but I didn’t throw the rock.”
The defense then offered the testimony of a
witness who was prepared to testify that she saw
the waiter throw the rock through the restaurant’s
window.
Assuming that there have been no criminal
charges filed as a result of the broken window, is
the witness’s testimony admissible?

(B) No, because specific acts of misconduct
that did not result in a conviction cannot be
established through extrinsic evidence.
(C) Yes, as evidence of bias.

A

(C) The witness’s testimony is admissible to show bias.

A witness can be impeached, either on cross-examination or by extrinsic evidence, with evidence that suggests a bias on the part of the witness, because it tends to show that the witness has a motive to lie. Evidence that the witness disliked the party he is testifying against would qualify as evidence of bias. The witness could testify that she saw the waiter throw the rock through the restaurant’s window, because such evidence would help establish the waiter’s bias against the restaurant.

(A) is incorrect for two reasons: Federal Rule 608
provides that, if offered to impeach, prior bad acts may not be proved through extrinsic evidence but may be inquired into during cross-examination. Furthermore, if the prior bad act also helps establish bias, the courts have held that extrinsic evidence also will be admissible. (B) is incorrect as well for this latter reason.

65
Q

At the defendant’s trial for rape, he calls a
witness who testifies that she was on her patio
barbecuing some hamburgers at the time of the
charged rape and saw the assailant run from the
victim’s apartment. She further testifies that the
person who ran from the victim’s apartment was
not the defendant.
On cross-examination by the prosecutor, to
which of the following questions would a defense
objection most likely be sustained?

(A) “Weren’t you convicted of perjury 11 years
ago?”

(D) “Didn’t you embezzle funds from your most
recent employer?”

A

(A) The objection to the perjury question is most likely to be sustained.

Federal Rule 609 permits the prosecution to inquire into prior convictions of crimes requiring proof or admission of dishonesty or false statement unless over 10 years have passed since the date of conviction or date of
release from confinement (whichever is later). While the facts do not indicate the latter date (or
even whether a confinement occurred), (A) remains the best of the four choices. The conviction in (A) is more than 10 years old, so it probably would be subject to objection as being too remote.

(D) relates to a prior bad act that shows dishonesty. Such acts may be asked about
on cross-examination of the witness.

66
Q

A plaintiff sued a defendant for serious personal injuries he incurred when the defendant allegedly drove through a red light and collided with the plaintiff’s car. Calling the defendant as an adverse witness, the plaintiff asked her if she had been drinking before the accident. The defendant refused to answer, asserting her privilege against self-incrimination. The plaintiff then offers in evidence a certified copy of a court record indicating that, eight years previously, the defendant had been convicted of reckless driving
while intoxicated that caused serious personal
injury, a felony. How should the trial court rule on the admissibility of the court record?

(A) Admit the record as relevant character evidence because the plaintiff suffered serious
personal injuries.
(B) Admit the record as impeachment evidence.
(C) Exclude the record as irrelevant because as
yet the defendant has given no testimony to
be impeached.
(D) Exclude the record because the conviction
is too remote and does not necessarily
reflect on the defendant’s credibility as a
witness in the present proceedings.

A

(C) The record of the conviction should be excluded because the defendant has given no testimony to be impeached.

Impeachment involves the casting of an adverse reflection on the truthfulness of a witness. Although the defendant has been called as a witness, she has not given any testimony at this point. Consequently, the plaintiff is unable to introduce evidence that would otherwise constitute proper impeachment evidence.

(A) is incorrect because evidence of character to prove the conduct of a person in the litigated event is generally not admissible in a civil case. Circumstantial use of prior behavior patterns for the purpose of inferring that, at the time and place in question, a person probably acted in accord with such patterns raises the danger of unfair prejudice and distraction from the main issues. Consequently, even if the prior conviction resulted from driving while intoxicated, the record of that conviction is not admissible to show that the defendant was intoxicated when she collided with the plaintiff.

(B) is incorrect because, as has been noted,
evidence cannot be used for impeachment purposes before there is anything to be impeached.

(D)
is incorrect for two reasons: First, it is unnecessary to address the issue of whether the conviction constitutes proper impeachment evidence, because impeachment is not even called for on these facts. Second, if properly offered to impeach testimony by the defendant, the conviction would not be considered too remote. Under the Federal Rules, a conviction is not too remote if fewer than 10 years have elapsed since the conviction or release from prison.

67
Q

A witness testified against a defendant in a contract action. The defendant then called the witness’s neighbor to the stand, who testified that the witness had a bad reputation for truth and veracity. The defendant then also called the witness’s employee to testify that the witness once perpetrated a hoax on an insurance company. The witness had in fact been convicted
for perpetrating the hoax 20 years ago. Is the employee’s testimony admissible?

(A) No, because it is merely cumulative impeachment.
(B) No, because it is extrinsic evidence of a
specific instance of misconduct.
(C) Yes, because the hoax resulted in a conviction
of the witness.

A

(B) The testimony is inadmissible because it is not a permitted way to impeach a witness.

A witness may be impeached by cross-examining her about specific criminal or immoral acts, but extrinsic evidence is not permitted. A specific act of misconduct offered to attack the witness’s character for truthfulness can be elicited only on cross-examination of the witness. If the witness denies it, the cross-examiner cannot refute the answer by calling other witnesses or producing other evidence. Thus, the witness could be asked on cross-examination about the hoax, but her employee cannot properly be called to testify about it.

(A) is incorrect because there is no specific rule limiting cumulative impeachment.

(C) is incorrect. A witness may also be impeached by introducing evidence that the witness was convicted of a crime if the conviction required proof or admission of an act of dishonesty or false statements, or if the crime was a felony. However, the prior conviction will generally not be admitted if more than 10 years have passed since the date of conviction or release from confinement, whichever is the later date (although the judge has discretion to
admit older convictions in extraordinary circumstances). Here the witness was convicted 20 years ago and the conviction is likely too remote. Additionally, the defendant did not attempt to introduce evidence of the witness’s conviction; the proposed testimony only concerns the commission of the misconduct.

68
Q

The plaintiff sued the defendant for injuries suffered when the defendant’s car struck the plaintiff as she was crossing a busy intersection. The plaintiff planned to have a bystander who had witnessed the accident testify on her behalf, but he died prior to trial. At trial, the plaintiff called the wife of the bystander to testify that,
although she had been facing the other way, she
had heard her husband exclaim, “My God, the
woman was crossing on the green light!” Over
objection, the statement was admitted as an
excited utterance. The defendant now wishes to
call the bystander’s friend, who is prepared to
testify that, a few hours after the accident, the
bystander said to him: “You know that accident
I saw this afternoon? The driver didn’t run a red
light. The light was yellow.” Should the friend’s testimony be admitted over
the plaintiff’s objection?

(A) Yes, but only to challenge the credibility of
the bystander’s earlier inconsistent statement.

(D) No, because it is hearsay, not within any
recognized exception to the hearsay rule.

A

(A) The friend’s testimony is admissible only to challenge the credibility of the bystander’s earlier
inconsistent statement.

Because the credibility of a hearsay declarant is as much at issue as the credibility of an in-court witness, Federal Rule 806 allows statements of a hearsay declarant to be impeached to the same extent as those of an in-court witness. Thus, a statement of the declarant made at any time that is inconsistent with his hearsay statement may be offered into evidence for impeachment purposes. Here, the bystander’s hearsay statement (which was admissible as an excited utterance) was testified to by his wife. His subsequent statement to his friend is inconsistent with his hearsay statement and is therefore admissible to discredit that statement.

(D) is wrong. While it is true that the statement is hearsay not within an exception (and thus inadmissible as substantive evidence), it is still admissible for the purpose of impeachment.

69
Q

The defendant was being sued by the plaintiff
for driving his car negligently and injuring the
plaintiff. The defendant called as a witness his
brother, who was in the passenger seat of the
defendant’s car when the accident occurred. His
brother testified that the defendant was driving
safely and well below the posted 55-mile-perhour
speed limit. Shortly before the trial began,
the brother’s secretary telephoned the plaintiff
and told him that the brother has been understating
his income to the government for years.
The Internal Revenue Service has never charged
the brother with tax evasion, but the secretary’s
information was accurate. On cross-examination,
the plaintiff’s attorney asked the brother, “Have
you ever cheated on your tax returns?” The
defendant’s attorney objects.
Should the objection be sustained?

A

The objection should be overruled because the question is a proper means of impeaching the brother’s character for truthfulness through specific instances of misconduct.

Under Federal Rule 608(b), subject to the discretion of the trial judge, a witness may be interrogated on cross-examination with respect to any specific act that may impeach his character and show him to be unworthy of belief, as long as the act is probative of truthfulness (i.e., an act of deceit or lying). A conviction of a crime is not necessary under this rule. Cheating on one’s taxes is lying, so this would be a specific act of misconduct reflecting on the brother’s character for truthfulness.

70
Q

The owner of a small business was injured in
a traffic accident. A month after the accident, the
owner asked an employee to take a photograph
of the intersection where the accident occurred.
The employee took the photograph and gave it
to the owner, who in turn gave it to his lawyer.
The lawyer wishes to introduce the photograph
into evidence at trial of the owner’s lawsuit
against the defendant. The lawyer plans to have
the employee testify that he took the photograph.
The lawyer also plans to call a witness who
lives in the neighborhood of the accident scene
and arrived at the intersection shortly after the
accident occurred. The witness is willing to
testify that the scene in the photograph is in fact
the intersection where the accident happened.
Whose testimony is necessary to introduce the
photograph into evidence?
(A) The employee’s testimony is necessary and
the witness’s is unnecessary.
(B) The witness’s testimony is necessary and
the employee’s is unnecessary.

A

(B) Only the witness’s testimony is necessary to introduce the photograph.

To be admissible, a photograph must be identified by a witness as a portrayal of certain facts relevant to the issue, and verified by the witness as a correct representation of those facts. It is sufficient if the witness who identifies the photograph is familiar with the scene or object depicted. It is not necessary to call the photographer to authenticate the photograph.

Here, the actual physical appearance of the
intersection is most likely relevant to the manner in which the accident occurred. As a resident of
the neighborhood in which the accident took place, and as someone who was at the scene of the accident shortly after its occurrence, the witness is sufficiently familiar with the scene to testify that the photograph is an accurate representation of the accident scene. Such identification by the witness is needed for the photograph to be admissible.

(A) incorrectly categorizes the employee’s
testimony as necessary. Generally, a photographer’s testimony is not necessary to authenticate a photo. In this case, it is particularly unhelpful because the employee is not familiar with the scene as it was when the accident occurred. Also, the testimony of the witness is necessary as a verification by one who is familiar with the scene.

71
Q

A plaintiff sued a defendant for breach of
contract and tried the case before a jury. The
parties disputed the existence and terms of
the contract. The plaintiff’s lawyer called the
plaintiff to the witness stand, intending to elicit
testimony that the plaintiff and the defendant
met on a certain date and reached an agreement,
which was reduced to a writing. The plaintiff
then intends to testify: “The writing, which was
subsequently destroyed, provided that the defendant would purchase 300 widgets from me at a price of $1,000.” What must be found for the quoted testimony to be admissible?

(A) The judge must find that the writing is unavailable, through no fault of the plaintiff.
(B) The judge must find that the writing is
unavailable, due to an intentional act by the
defendant.

A

(A) The judge must find that the writing is unavailable, through no fault of the plaintiff.

Under the best evidence rule, when a document is unavailable but is central to the resolution of a dispute (i.e., not collateral) and there is an attempt to enter the contents of that document into evidence through testimony, the judge must first find that the document itself is not available through no fault of the proponent (here, the plaintiff), in order for testimony on the issue to be admissible.

(B) is wrong because the proponent need not prove that an opposing party contributed to the loss of the document.

72
Q

Generally, the physician-patient privilege provides that a physician cannot be compelled to disclose information obtained from a patient while treating the patient in a professional capacity if that information is related to the treatment. The privilege does not, however, apply in

A

…. federal cases where state law does not supply the rule of privilege (i.e., federal question cases).

73
Q

A driver and his passenger were involved in
an accident with another car. They consulted
a lawyer regarding their claim and decided
to file a suit against the other driver. The
lawyer’s paralegal attended the consultation.
The passenger subsequently sued the driver on
a cross-complaint. Prior to trial, the paralegal
left the lawyer’s employ. At trial, the driver’s
lawyer called his former employee, the paralegal,
to take the stand. The paralegal will testify
regarding the passenger’s statements to the
lawyer at the initial consultation that he was
arguing with the driver at the time of the collision.
Is the testimony admissible?
(A) Yes, because the paralegal’s presence destroyed the attorney-client privilege.
(B) Yes, as a statement by an opposing party.
(C) No, because it is protected by the attorneyclient privilege.

A

(B) The statement of the passenger is admissible as a statement by an opposing party. A statement
made by a party and offered against that party (commonly called an admission) is admissible
nonhearsay. Here, the driver’s lawyer is seeking to admit the passenger’s statement that he was
arguing with the driver when the collision occurred. The testimony relates to the issue of fault and is admissible. (A) and (C) are incorrect. The attorney-client privilege protects from disclosure communications between a client and an attorney made during professional consultation. The privilege applies even if the communication is made in the presence of an agent of the attorney. The privilege does not apply, however, in suits between two parties represented by the same attorney. Here, the privilege would apply despite the paralegal’s presence because she is an agent of the lawyer. It does not, however, apply in the suit between the driver and passenger because they were represented by the same attorney.

74
Q

A plaintiff was injured in an automobile
accident caused by the defendant. The plaintiff
sued the defendant for his injuries. In preparation
for trial, the plaintiff’s attorney hired a doctor
to examine the plaintiff. At trial, the defense
attorney attempts to call the doctor as a witness
to testify about statements the plaintiff made in
confidence to the doctor about his injuries, which
the doctor then communicated to the plaintiff’s
attorney. The state recognizes only the common
law privileges.
Should this testimony be admitted?

(B) Yes, because the plaintiff waived the
physician-patient privilege by placing his
physical condition in issue.
(C) No, because the plaintiff’s statements are
protected by the attorney-client privilege.

A

(C) The testimony should be excluded because the attorney-client privilege applies to the examination done in preparation for trial.

The communication between the doctor and the attorney’s client is necessary to help the client convey his condition to the attorney.

(B) is a true statement; the physician-patient privilege does not apply to any proceeding in which the condition of the patient has been put in issue by the patient. However, (B) is incorrect because when a client is examined by a doctor at the attorney’s request, the communications involved between the client and doctor (and the doctor and attorney) are not covered by the physician-patient privilege because no treatment is contemplated. Moreover, the physician-patient privilege is a statutory privilege, and this jurisdiction recognizes only the common law privileges (e.g., the attorney-client privilege).

75
Q

A defendant was visiting with his girlfriend
in his apartment when a visitor came to see
him. The defendant and the visitor engaged in
a conversation relating to the distribution of
illegal narcotics in the girlfriend’s presence. Two
months later, the defendant and his girlfriend
married. Subsequent to the marriage, the defendant was arrested and charged under federal law with the sale and distribution of drugs. The prosecutor wants the defendant’s wife to testify about the conversation between the defendant and the visitor, but the defendant forbids it. May the defendant’s wife testify about the conversation?

(B) Yes, but only if she chooses to do so.
(C) No, because the defendant forbids it.

A

(B) The wife may testify if she chooses to do so.

In federal court, the privilege of spousal immunity
belongs to the witness-spouse.

There are two privileges based on the marital relationship.

Under spousal immunity, a person whose spouse is the defendant in a criminal case may not be called as a witness by the prosecution, and a married person may not be compelled to testify against her spouse in any criminal proceeding. In federal court, one spouse may choose to testify against the other in a criminal case, with or without the consent of the party-spouse. Spousal immunity lasts only during the marriage and terminates upon divorce. However, as long as a marriage exists, the privilege can be asserted even as to matters that occurred prior to the marriage. Because the defendant is a criminal defendant, his wife cannot be compelled to testify about his conversation
with the visitor. She may, however, choose to testify, and the defendant cannot stop her.

Under confidential marital communications privilege, either spouse (whether or not a party) may refuse to disclose, and may prevent another from disclosing, a confidential _communication made between the spouses_ while they were husband and wife. The communication must be made during a marriage, and must be in reliance upon the intimacy of the marital relationship, which is presumed in the absence of contrary evidence. This privilege is not afforded to a communication that is made in the known presence of a stranger. Both spouses jointly hold this privilege.

The conversation between the defendant and the visitor cannot qualify as a confidential marital communication for several reasons. Most importantly, it was not a communication between the defendant and his wife. Moreover, the incident did not occur during the marriage. Thus, the privilege for confidential marital communications does not apply, and the defendant cannot prevent his wife’s testimony should she choose to testify.

(C) is wrong because, in federal court, spousal
immunity does not permit the defendant-spouse to foreclose testimony by the witness-spouse. As
discussed above, the privilege for confidential marital communications, under which both spouses may prevent disclosure, does not apply here.

76
Q

A defendant was charged with murder. After
he was arrested, he admitted to police detectives
after several hours of questioning that he killed
the victim. Before trial commences, the defense
attorney requests a preliminary determination
as to whether the defendant’s confession was
voluntary.
Under what circumstances will the voluntariness
of the confession be determined?
(A) The jury will make the determination in the
presence of the judge.
(C) The judge will make the determination, and
she must excuse the jury from the hearing.

A

(C) The judge will make the determination and the jury must be excused.

Whether the jury should be excused during a preliminary fact determination is generally within the discretion of the trial judge. However, because of the potential for prejudice to the accused in a criminal trial, Federal Rule 104(c) provides that the jury must be excused during hearings on the admissibility of the accused’s confession.

(A) is incorrect. The judge, not the jury, must decide any preliminary question about whether evidence—such as a confession—is admissible.

77
Q

At trial, questions have been raised as to
whether the proposed testimony of the witness
is relevant and whether it falls within the present
sense impression exception to the hearsay rule.
How should a preliminary determination of
the admissibility of the witness’s testimony be
made?
(A) A judge should determine whether the proposed testimony falls within the exception
before it is heard by the jury, and in making
that determination she is limited by the
rules of evidence.

(B) A judge should decide whether the testimony
falls within the present sense impression exception, but in making that determination she is not limited by the rules of evidence other than privilege.

A

(B) The judge determines whether the testimony falls within an exception to the hearsay rule, and is generally not limited by the rules of evidence in making that determination.

The Federal Rules of Evidence distinguish between preliminary facts to be decided by the jury, which involve whether the proffered evidence is relevant, and preliminary facts decided by the judge, which involve whether the evidence is competent, i.e., not barred by an exclusionary rule.

All preliminary fact questions that determine the applicability of an exception to the hearsay rule must be determined by the judge, because the competency of the evidence will depend on that preliminary fact determination. In making this preliminary fact determination, the trial court may consider any nonprivileged relevant evidence, even though it would not otherwise be admissible under the rules of evidence. [Fed. R. Evid. 104(a)]

In this case, then, the judge should decide whether the testimony falls within the present sense impression exception, and she is not limited in making this determination by the rules of evidence other than privilege.

78
Q

The plaintiff sued the defendant, alleging that
the defendant allowed her dogs to roam onto the
plaintiff’s land and cause significant damage
to his landscaping. The defendant denied the
allegations and called a witness to testify on her
behalf. The witness testified on direct examination that she visited the defendant every day and that the defendant never allowed her dogs to leave the perimeter of her property. On crossexamination, the plaintiff’s counsel presented the witness with a letter written by the witness to a friend in which she expressed her dismay that the defendant allowed her dogs to roam throughout the neighborhood. The plaintiff’s counsel requests that the witness read the letter to herself prior to cross-examination. The defendant objects. May the witness refer to the letter?

(C) No, because it has not been formally
offered into evidence.
(D) No, because the witness has not claimed
any inability to remember.

A

(D) The witness may not refer to the letter.

Generally, a memorandum can be used to refresh the recollection of a witness, to substitute for forgotten testimony, or on cross-examination.

Here, the plaintiff’s counsel can use the letter to impeach the witness’s testimony because it is a prior inconsistent statement. To do so, he can cross-examine her as to the contents of the letter (as opposed to having her read it to herself prior to cross-examination, as he attempted to do) to show that she has made statements contrary to her testimony, thereby discrediting her testimony.

(C) is incorrect because the memorandum does not need to be offered into evidence; a witness may simply refer to it to refresh recollection prior to giving her testimony.

79
Q

In which of the following cases would an
objection to a leading question most likely be
upheld?

(A) When asked on direct examination of a
disinterested witness.

(C) When asked on cross-examination of an
expert witness.

A

(A) Leading questions are allowed on the direct examination of a “hostile” witness.

There is no rule that allows leading questions on the direct examination of a “disinterested” witness.

A leading question is normally permitted on cross-examination whether the witness is a layperson or an expert. Therefore, (C) is wrong.

80
Q

There is a hearsay exception that allows portions of learned treatises to be read into evidence, but this exception applies only when the treatise is being used on direct or cross-examination of an expert witness.

A

i.e.

Here the prosecutor is trying to read portions of the treatise before calling any expert witness to the stand, so the treatise is inadmissible hearsay.

81
Q

A police officer stopped a driver who had run a red light. Upon approaching the car, the officer noticed a strong odor of alcohol and immediately asked whether the driver had been drinking. The driver admitted having had several alcoholic drinks that evening.

The driver, charged with driving while intoxicated, moved to suppress the officer’s testimony regarding the driver’s statement about his drinking. The driver argued that the officer had elicited the statement without providing the requisite Miranda warnings. The prosecutor has responded that the statement should be allowed in the prosecution’s case-in-chief or, at a minimum, should be allowed as impeachment in the event the driver testifies and denies drinking.

How should the court rule regarding the driver’s statement admitting his drinking?

A. The statement should be allowed, because although the driver was in custody, the officer’s spontaneous utterance upon smelling alcohol did not rise to the level of interrogation.

B. The statement should be allowed, because the driver was not in custody for Miranda purposes when the admission was made.

C. The statement should be suppressed both in the prosecution’s case-in-chief and as impeachment evidence, even if the driver testifies.

D. The statement should be suppressed in the prosecution’s case-in-chief, but it may be used as impeachment evidence if the driver testifies.

A

(B) is correct.

As a general rule, to offset the coercive nature of custodial interrogation by a police officer and protect defendants’ Fifth Amendment right against compelled self-incrimination, the Supreme Court has made Miranda warnings a prerequisite to the admissibility of confessions obtained during a custodial police interrogation. The officer’s question here was an interrogation, as the term “interrogation” extends to any words or conduct used by an officer intentionally to elicit an incriminating response. Nevertheless, the driver’s response here is admissible despite the lack of Miranda warnings because the driver was not in custody. Determining whether a person is in custody for Miranda purposes involves a two-step inquiry. The Court will first look to whether a reasonable person in the circumstances would feel free to terminate the encounter and leave. If not, the Court looks to the whether the relevant environment presents coercive pressures similar to those of a station house interrogation. The more a setting resembles a traditional arrest, the more likely the Court will find the person to be in custody.

Here, the driver, no doubt, felt that he was not free to leave, having just been pulled over for a traffic violation. However, a traffic stop does not resemble a traditional arrest. Given their brief nature and the motorist’s knowledge that he will soon be on his way, the Supreme Court has found traffic stops to be noncustodial. Therefore, a police officer can ask questions during a routine traffic stop without giving Miranda warnings, and responses to those questions are admissible at trial in a case-in-chief and for impeachment purposes. Therefore, (A), (C), and (D) are incorrect.

82
Q

Under the former testimony exception to the hearsay rule, the testimony of a now unavailable witness given at another hearing is admissible in a subsequent trial as long as there is a sufficient similarity of parties and issues so that the opportunity to develop testimony or cross-examine at the prior hearing was meaningful. [Fed. R. Evid. 804(b)(1)] The party against whom the former testimony is offered must have had the opportunity to develop the testimony at the prior proceeding by direct, cross-, or redirect examination of the declarant.

Is the grand jury testimony of an unavailable declarant admissible as former testimony against the accused at trial?

A

No because grand jury proceedings do not provide the opportunity for cross-examination.

83
Q

Any person can testify to the authenticity of another’s signature as long as that witness has seen the person’s signature and can express an opinion regarding its authenticity. The only restriction is that …

A

… a nonexpert cannot become familiar with the handwriting merely for the purpose of testifying. There is no requirement that the witness have seen the signature recently.

84
Q

Lay opinion testimony is admissible when:

A

(i) it is rationally based on the perception of the witness;
(ii) it is helpful to a clear understanding of his testimony or to the determination of a fact in issue;

and (iii) it is not based on scientific, technical, or other specialized knowledge. [Fed. R. Evid. 701]

85
Q

statements regarding past symptoms and medical history made to assist in diagnosis or treatment are admissible, even if made to

A

a doctor employed to testify

86
Q

An admission is a statement made by a party and offered against that party.

A

A plea of guilty to a traffic infraction is a formal judicial admission. The admission is conclusive in a prosecution for that infraction, but if the plea is used in another proceeding, it is merely an evidentiary admission (i.e., it is not conclusive and can be explained).

Here, the defendant has acknowledged by her guilty plea that she was speeding at the time of the accident. This fact is relevant to the plaintiff’s suit for personal injuries because it increases the likelihood that the defendant was at fault in the accident that caused those injuries. Therefore, the defendant’s guilty plea is admissible in the current civil action as an evidentiary admission.

87
Q

A homeowner returned home from work one day to find a robber in her living room. After a brief physical altercation, the homeowner ran to a bedroom, hid in a closet, and called 911 on her cell phone. Police officers arrived in less than two minutes and were able to apprehend the robber as he tried to run out the front door. Once they made sure he was locked in the police car, one of the officers went to speak with the homeowner about what had happened. She was still crying and shaking when the officer found her, and she said, “Thank you for catching him! He punched me in the head as I was running away!” The robber was charged with robbery and assault. Traumatized, the homeowner left the country and cannot be traced, despite the efforts of the prosecutor. The prosecutor intends to call the officer to testify as to the homeowner’s statement.

Should the court allow the officer’s testimony?

A Yes, because the homeowner’s statement is an excited utterance.

B Yes, because the homeowner’s statement is not testimonial.

C No, because admitting the homeowner’s statement would violate the defendant’s constitutional rights.

D No, because the homeowner’s statement is hearsay not within any exception.

A

The testimony should not be admitted. Under the Confrontation Clause, an accused has the right to be confronted by the witnesses against him.

A hearsay statement will not be admitted—even if it falls within a hearsay exception—when:

(i) the statement is offered against the accused in a criminal case;

(ii) the declarant is unavailable;

(iii) the statement was testimonial in nature; and

(iv) the accused had no opportunity to cross-examine the declarant’s “testimonial” statement prior to trial.

The Supreme Court has established that if the primary purpose of police interrogation is to enable the police to help in an ongoing emergency, statements made in the course of the interrogation are nontestimonial. When the primary purpose of the interrogation is to establish or prove past events potentially relevant to a later criminal prosecution, statements are testimonial.

Here, the homeowner’s statement was hearsay because it was made out of court and is being offered for its truth—that the robber punched her in the head. It appears that the emergency had already resolved by the time the statement was made. Although only a few minutes had passed since the physical altercation and the homeowner was still upset, the robber no longer posed any danger because he was locked in the police car and the homeowner was aware of this (“Thank you for catching him!”). Therefore, the homeowner’s statements to the officer were testimonial. Because the homeowner is unavailable to testify at trial and the robber has had no opportunity to cross-examine the statements, admitting them at trial through the testimony of the officer would violate the Confrontation Clause.

(A) is incorrect. Under the excited utterance exception, a declaration made by a declarant during or soon after a startling event is admissible if it relates to the startling occurrence and was made under the stress of excitement produced by the startling event. Here, finding a robber in one’s home is a startling event, and the homeowner was still under the stress of the excitement when she made the statement—she was crying and shaking. The statement likely qualifies as an excited utterance. However, even a hearsay statement that falls within an exception may be barred by the Confrontation Clause, as is the case here. (B) is incorrect. As stated above, when the primary purpose of the interrogation is to establish or prove past events potentially relevant to a later criminal prosecution, statements are testimonial. Because the homeowner’s statement was made for this purpose and not to address an ongoing emergency, it was testimonial in nature and its admission would violate by the Confrontation Clause. (D) is incorrect. As stated above, the homeowner’s statement falls within the exception for excited utterances, so it will not be excluded because of the hearsay rule. The testimony will be excluded because of the robber’s constitutional right to be confronted by the witnesses against him.

88
Q

evidence in the form of a certification or testimony from the custodian of public records that she has diligently searched and failed to find a record is admissible to prove that a matter was not recorded, or, inferentially, that …

A

a matter did not occur

89
Q

A pedestrian sued a driver after the driver’s car hit the pedestrian at an intersection. The pedestrian claims that the driver ran a stop sign and the driver contends that the pedestrian darted out into traffic. The pedestrian seeks to call her friend to the stand to testify that the pedestrian is an extremely cautious person who invariably obeys traffic laws.

Should the judge admit the friend’s testimony?

A Yes, because it is proper habit evidence.

B Yes, because it tends to show that the pedestrian is less likely to have darted out into traffic.

C No, because the pedestrian’s character is not directly in issue.

D No, because the friend is a biased witness.

A

This is character evidence because it describes the pedestrian’s general tendency to act cautiously. Evidence of a person’s character is generally irrelevant and inadmissible in a civil case except in rare circumstances where character is directly in issue (e.g., defamation or negligent hiring cases). Here, because the pedestrian’s character is not directly in issue in this civil personal injury lawsuit, the friend’s testimony is inadmissible. (A) is incorrect. Under Federal Rule 406, evidence of a person’s habit may be admitted to prove that on a particular occasion the person acted in accordance with the habit. Habit describes one’s regular response to a specific set of circumstances, while character describes one’s disposition in respect to general traits. Since habits are more specific and particularized, evidence of habit is relevant and can be introduced in circumstances when it is not permissible to introduce evidence of character. Here, however, the friend’s testimony is character evidence, not habit evidence. The fact that the pedestrian is cautious and obeys traffic laws is not nearly specific enough to qualify as habit evidence—despite the use of the term “invariably.” (B) is incorrect. The answer choice recites the general rule of relevance—that is, evidence is relevant if it makes the existence of any fact of consequence to the determination of the action more likely or less likely than it would be without the evidence. However, despite meeting the low threshold for relevance, character evidence is generally inadmissible in civil cases to prove that a person acted in conformity with the trait during the events of the case. (D) is incorrect. Although the credibility of a witness may be impeached by showing that the witness is biased, that does not mean the testimony itself is inadmissible. Indeed, most witnesses testifying as to the habit or character of a person will have some sort of personal bias in favor of or against that person.

90
Q

The opinions of experts who are retained in anticipation of litigation but who are not expected to testify at trial may be discovered

A

only upon a showing of exceptional circumstances under which it is impracticable to obtain facts or opinions by other means or when a medical report for an exam conducted under FRCP 35 is requested.

91
Q

A plaintiff was injured in an automobile accident caused by the defendant. The plaintiff sued the defendant for his injuries. In preparation for trial, the plaintiff’s attorney hired a doctor to examine the plaintiff. At trial, the defense attorney attempts to call the doctor as a witness to testify about statements the plaintiff made in confidence to the doctor about his injuries, which the doctor then communicated to the plaintiff’s attorney. The state recognizes only the common law privileges.

Should this testimony be admitted?

A Yes, because the plaintiff’s statements are the statements of a party-opponent.

B Yes, because the plaintiff waived the physician-patient privilege by placing his physical condition in issue.

C No, because the plaintiff’s statements are protected by the attorney-client privilege.

D No, because the plaintiff’s statements are protected by the physician-patient privilege.

A

The testimony should be excluded because the attorney-client privilege applies to the examination done in preparation for trial. The communication between the doctor and the attorney’s client is necessary to help the client convey his condition to the attorney.

(A) is incorrect because admissions by party-opponents, while not hearsay under the Federal Rules, are still subject to potential privilege assertions.

(B) is a true statement; the physician-patient privilege does not apply to any proceeding in which the condition of the patient has been put in issue by the patient. This is the case in the plaintiff’s suit, so (D) is incorrect. *However, (B) is incorrect because when a client is examined by a doctor at the attorney’s request, the communications involved between the client and doctor (and the doctor and attorney) are not covered by the physician-patient privilege because no treatment is contemplated. Moreover, the physician-patient privilege is a statutory privilege, and this jurisdiction recognizes only the common law privileges (e.g., the attorney-client privilege).*

92
Q

At trial, questions have been raised as to whether the proposed testimony of the witness is relevant and whether it falls within the present sense impression exception to the hearsay rule.

How should a preliminary determination of the admissibility of the witness’s testimony be made?

A A judge should determine whether the proposed testimony falls within the exception before it is heard by the jury, and in making that determination she is limited by the rules of evidence.

B A judge should decide whether the testimony falls within the present sense impression exception, but in making that determination she is not limited by the rules of evidence other than privilege.

C The jury, after being instructed on the rules of evidence by a judge, should determine whether the testimony falls within the scope of the present sense impression exception.

D The jury should determine whether the testimony falls within the scope of the exception and the judge should then instruct the jury on the appropriate uses for that evidence.

A

The judge determines whether the testimony falls within an exception to the hearsay rule, and is generally not limited by the rules of evidence in making that determination.

The Federal Rules of Evidence distinguish between preliminary facts to be decided by the jury, which involve whether the proffered evidence is relevant, and preliminary facts decided by the judge, which involve whether the evidence is competent, i.e., not barred by an exclusionary rule. All preliminary fact questions that determine the applicability of an exception to the hearsay rule must be determined by the judge, because the competency of the evidence will depend on that preliminary fact determination. In making this preliminary fact determination, the trial court may consider any nonprivileged relevant evidence, even though it would not otherwise be admissible under the rules of evidence. [Fed. R. Evid. 104(a)] In this case, then, the judge should decide whether the testimony falls within the present sense impression exception, and she is not limited in making this determination by the rules of evidence other than privilege. (A) is incorrect because the judge’s preliminary fact determination does not need to be based on the rules of evidence (other than privilege rules). (C) and (D) are incorrect because, as discussed above, determining whether the testimony falls within the scope of the hearsay exception is a determination of whether the evidence is competent, and this determination is made by the judge rather than the jury.

93
Q

A plaintiff sued a defendant over a claimed debt. At the trial, the plaintiff established the existence of the debt and testified that he never received payment. In response, the defendant presents evidence sufficient to establish that she took her check to the post office and sent it to the plaintiff’s proper address by certified mail. The defendant offers a certified mail receipt with an illegible signature, which she claims is the plaintiff’s signature. The defendant also presents evidence that her basement flooded on March 28, and she claims that she cannot produce a canceled check because her box of canceled checks was destroyed from the water damage. Evidence is also presented that, due to a computer glitch, the defendant’s bank cannot reproduce her checking account records for the months of February and March.

After the defendant’s testimony, which of the following is correct?

A The burden of persuasion and the burden of going forward with the evidence are on the plaintiff.

B The burden of persuasion is on the plaintiff, but he has no burden of going forward with the evidence.

C The plaintiff has satisfied his burden of persuasion, but he has a burden of going forward with the evidence.

D The plaintiff has satisfied both his burden of persuasion and his burden of going forward with the evidence.

A

The burden of persuasion and the burden of going forward with the evidence are on the plaintiff because the defendant’s testimony raises a rebuttable presumption that the check had been delivered in the mail.

The burden of persuasion is the burden of a party to persuade the jury to decide an issue in its favor. If, after all the proof is in, the issue is equally balanced in the mind of the jury, then the party with the burden of persuasion must lose. The burden of persuasion does not shift from party to party during the course of a trial. Because the plaintiff sued the defendant for the debt, the plaintiff has the burden of persuasion when the time for the jury to make a decision arrives.

The burden of going forward with the evidence is the burden of producing sufficient evidence to create a fact question of the issue involved.

If a plaintiff makes out a prima facie case, he has met his burden of going forward with the evidence and the burden shifts to the defendant.

Here, when the plaintiff made out a prima facie case of the defendant’s debt, the burden of going forward with the evidence shifted to the defendant. The defendant met this burden through the use of a presumption.

Federal Rule 301 provides that a presumption imposes on the party against whom it was directed the burden of going forward with the evidence to rebut the presumption.

The defendant’s evidence regarding the proper posting of the check raises a rebuttable presumption that the check was delivered to the plaintiff because a letter shown to have been properly addressed, stamped, and mailed is presumed to have been delivered in the due course of mail. Therefore, the burden of going forward with the evidence has shifted back again to the plaintiff, who must now produce evidence to rebut the presumption (i.e., evidence that he did not receive the check).

(B) is incorrect because, as discussed above, the defendant’s testimony raised a rebuttable presumption that the check was delivered in the mail, which shifted the burden of going forward with the evidence to the plaintiff. The fact that the plaintiff met his burden of going forward with the evidence of the debt once, when he made out his prima facie case, does not mean the burden cannot shift back to him. (C) is incorrect because the plaintiff has not satisfied his burden of persuasion. As discussed above, the burden of persuasion does not shift from party to party and is only a crucial factor when all the evidence is in. This burden is satisfied when the jury finds a party has been more persuasive in arguing his side of the issue than the other party. Because the defendant’s testimony raises a rebuttable presumption that the check was delivered to the plaintiff, the plaintiff’s burden of persuasion cannot be met until he offers evidence to prove that the check was not received (a necessary element of his case). (D) is incorrect because, as discussed above, the defendant’s testimony raised a rebuttable presumption of delivery of the check in the mail to the plaintiff, which shifted the burden of going forward with evidence of nondelivery back to the plaintiff. The plaintiff’s burden of persuasion cannot be satisfied until he comes forward with this evidence because a necessary element of his case is that the defendant never paid him.

94
Q

In litigation over whether an uncle conveyed a parcel of land to his nephew, the nephew wishes to offer into evidence a tape recording of his uncle made by a well-known oral historian at the nearby state university. The voice on the tape is discussing various conveyances of the parcel of land and other property owned by the uncle. The nephew wishes to have the historian testify that the voice on the tape is the uncle’s.

If the court allows the historian to testify, it will be because:

B The historian has heard the uncle speak before.

C The historian became familiar with the uncle’s voice before the dispute over the property arose.

A

The court will allow the historian to testify as to the identity of the voice simply because he is familiar with the uncle’s voice.

Where the identity of a speaker is important, the oral statements require authentication as to the identity of the speaker. A voice, whether heard firsthand or through a tape recording, may be identified by the opinion of anyone who has heard the voice at any time. As long as such a foundation is laid to show familiarity with the voice, a lay opinion as to the identity of the speaker is permissible. Thus, because the historian became familiar with the uncle’s voice when he made the tape recording, he will be permitted to testify that the voice on the tape was the uncle’s.

(C) is incorrect because, in contrast to the rule for handwriting verification, a person can become familiar with a voice after litigation has begun and for the sole purpose of testifying. Hence, the fact that the historian became familiar with the uncle’s voice before the dispute arose is not critical to admissibility of his testimony.

95
Q

Prior inconsistent statements are admissible as substantive evidence (and not excluded as hearsay) under the Federal Rules of Evidence [Fed. R. Evid. 801] if …

A

… the declarant testifies at the trial and is subject to cross-examination, and the prior statement was given under oath at a prior proceeding.

If they do not meet these standards, prior inconsistent statements are hearsay that is admissible for the limited purpose of impeachment.

96
Q

A trial court’s ruling on discretionary matters, such as the admissibility of evidence, is subject to review under

A

the abuse of discretion standard.

97
Q

A trial judge presiding over a lawsuit plans to call a witness to the stand and question her.

May the judge do so?

A Yes, but only if the witness has already been called and examined by one of the parties.

B Yes, but only if the witness is not testifying as an expert.

C Yes, but the parties are entitled to cross-examine the witness.

D No, because only parties may call and examine witnesses.

A

The judge may call and examine the witness, but the parties are entitled to cross-examine the witness. A court is entitled to examine any witness called by any party, and may also call a witness on its own or at a party’s request. Therefore, (D) is wrong. (A) is wrong because the court may call a witness on its own, even if the witness has not been called by any of the parties. (B) is wrong because the court may call any witness, even an expert. Although the court has wide discretion to call and examine witnesses, each party is entitled to cross-examine the court’s witness. A party may also object to the court’s examining or calling a witness either at that time or at the next opportunity when the jury is not present (to spare counsel the potential embarrassment of objecting to the judge’s questions in front of the jury).

98
Q

Under the Federal Rules of Evidence, most presumptions can be rebutted by evidence that contradicts the presumption.

A

If one party lays a proper foundation for the presumption and the other party does not introduce any contrary evidence, the judge will instruct the jury on the presumption.

If the other party produces even a scintilla of credible evidence contradicting the presumption, the judge will not instruct the jury on the presumption.

i.e.

The attorney claims that she mailed the letter to the law firm, but the law firm denies receiving it. Given that the law firm has produced testimonial evidence contradicting the presumption, the judge should not instruct the jury on the presumption.

*

Evidence state that a conclusive presumption is one which the jury must accept no matter what evidence to the contrary is introduced. Conclusive presumptions are rare.

99
Q

The shopper’s statement that he had serious back pain before the slip and fall accident in the store.

statement by a party-opponent

or

statement against interest?

A

The shopper’s statements to the witness are admissible as statements by a party-opponent. These are words or acts of a party-opponent offered as evidence against him.

Under Federal Rule of Evidence 804(b)(3), a statement against interest requires that:

(1) the declarant is unavailable at trial;

(2) at the time the statement was made, it was contrary to the declarant’s pecuniary, proprietary, or penal interest, or tended to subject the declarant to civil or criminal liability; and
(3) a reasonable person in the declarant’s position would not have made the statement if it were not true.

The shopper is available at trial, and the statement was probably not against any interest at the time he made it, so this exception does not fit. Note that this hearsay exception will not be the correct answer when the declarant is a party to the action; opposing party’s statement answers are better in that type of fact pattern. Thus, this answer is incorrect.

100
Q

Confrontation Clause

Examples of out-of-court “testimonial statements”

A

In a criminal case, the Sixth Amendment requires the opportunity to cross-examine testimonial hearsay statements.

Examples of out-of-court “testimonial statements” include ones made to law enforcement officers, other government employees or officials, and statements made in courtrooms or courtroom-like settings or other analogous situations. This includes prior testimony in a preliminary hearing, before a grand jury, at a prior trial and also police interrogation situations. Affidavits and depositions are also viewed as being testimonial statements. Basically, the common thread among statements made in such environments is the more “official” or formal nature of such communications.

In contrast, hearsay statements to family, friends, co-workers, and neighbors, which are typically informal and conversational in nature, are not considered testimonial. Calls to 911 and similar statements to law enforcement which, when viewed objectively, have as their primary purpose to enable law enforcement to respond to an ongoing emergency are not testimonial in nature.

101
Q

Is question on cross-examination containing impeachment BICCC beyond the scope of direct examination when direct examination was only about preliminary matters?

A

Any witness may be questioned during cross-examination about acts of misconduct when they relate to the witness’s credibility. Credibility is in issue for all witnesses, and so impeachment on the grounds of their ability to perceive or recall is always acceptable. Similarly, bias and tendency toward truth-telling are always relevant as to the credibility of a witness who takes the stand to testify.

Any witness may be questioned during cross-examination about acts of misconduct when they relate to the witness’s credibility. Therefore, this is within the scope of direct examination.

102
Q

The question involves the admissibility of settlement offers (“I’ll settle for a refund of the purchase price plus $50,000”) and statements made in the context of settlement negotiations (“Well, maybe it wasn’t fraud”).

Admissible?

A

Both settlement offers and statements made during settlement negotiations are inadmissible. [Fed. R. Evid. 408]

This is because public policy favors the voluntary settlement of disputes. If a settlement offer has been made in a case that ultimately goes to trial, admission of the offer into evidence would probably prejudice the party who made the offer. Thus, if such evidence were admissible, there would be a disincentive to parties making settlement offers. Similarly, if statements made during settlement negotiations could be admitted into evidence in cases that ultimately go to trial, then that would be a disincentive to enter into settlement negotiations or to discuss settlement in a freewheeling way. In either event, the public policy favoring settlements bars use of such statements.

103
Q

There is no hearsay exception for out-of-court statements given under hypnosis. The only hearsay exception that seems possibly applicable is the residual or “catch-all” exception, Federal Rule 803(24). Before evidence can be admitted under this exception, however,

A

an elaborate pretrial notice procedure must be followed to assure that the other party has an opportunity to prepare to meet the hearsay evidence.

104
Q

At a products liability trial, a critical issue is whether the temperature was below freezing on January 16. A local man who works for a civil engineering firm is also an avid amateur meteorologist. One of the man’s weather detection instruments in his backyard records temperature by markings from a stylus on a round barograph. The man’s record of the day in question indicates that it was unseasonably warm and that the temperature never fell lower than 48 degrees Fahrenheit, 16 degrees above the freezing mark. The plaintiff offers into evidence the man’s barograph record of the temperature on January 16.

Is the barograph record admissible?

A Yes, as past recollection recorded.

B Yes, as a record of regularly conducted activity.

C No, unless foundation testimony is given as to the accuracy and good working order of the man’s instrument on the day in question.

D No, because it is hearsay not within any recognized exception to the hearsay rule.

A

The barograph record is admissible only if it was properly authenticated.

Before a writing or any secondary evidence of its content may be received in evidence, the writing must be authenticated by proof showing that the writing is what the proponent says it is.

In general, a writing may be authenticated by any evidence that serves to establish its authenticity.

One means of authentication under Federal Rule 901(b) is by evidence describing a process or system used to produce a result and showing that the process or system produces an accurate result. Hence, for the man’s barograph record to be admissible, evidence must have been offered that the instrument is accurate and that it was in good working order when the record was produced.

105
Q

The Federal Rules conform to the existing state rules governing judicial notice. Federal Rule 201(b) defines a fact that may be noticed as “one not subject to reasonable dispute in that it is either (i) generally known within the territorial jurisdiction of the trial court, or (ii) capable of accurate and ready determination by resort to sources whose accuracy cannot reasonably be questioned.” To be considered generally known within the community, the fact must be something that well-informed people generally know and accept.

A

Although usually facts of common knowledge are known everywhere, it is sufficient for judicial notice if they are known in the community where the court is sitting.

106
Q

The plaintiff instituted suit against a development firm and its employee for injuries suffered by the plaintiff when a basement stair collapsed as he was being shown one of the firm’s buildings that it was leasing. At trial, the employee for the firm claims that he told the plaintiff about the defective stair. The plaintiff then offers evidence that after his fall, the CEO of the development firm called him at home and offered to pay all of the plaintiff’s medical expenses, saying, “I guess I owe you that much after our employee didn’t warn you about the broken stair.”

The statement regarding the lack of warning is:

A Admissible, as a statement against interest.

B Admissible, as an admission by the CEO of the development firm that the employee had not warned the plaintiff about the defective stair.

C Inadmissible, as hearsay not within any exception.

D Inadmissible, on public policy grounds.

A

The offer to pay medical expenses in itself is not admissible as an admission. Here, however, there is a statement that follows such an offer. This statement is admissible as an admission by the CEO of the development firm that the employee had not warned the plaintiff about the dangerous condition of the stairs. Because this is an admission, it is not hearsay under the Federal Rules. (A) is an exception to the hearsay rule and is thus unnecessary. Furthermore, it is an exception requiring the unavailability of the declarant, and there is no indication in the facts that the employee is now unavailable. (C) is wrong because, as mentioned, the statement is not hearsay under the Federal Rules. (D) is a misstatement of the law; only the offer to pay medical expenses is inadmissible on public policy grounds.

107
Q

The plaintiff sued the defendant, who had constructed the plaintiff’s house, for breach of warranty of habitability. At trial, in cross-examination of the plaintiff, the defendant’s attorney asked whether the plaintiff had sued another contractor 30 years earlier, claiming similar defects in another house built for the plaintiff. The question was not objected to and the plaintiff answered that she had had some “water problems” with the first house she ever purchased, but no suit was filed.

The defendant then called as a witness the contractor of 30 years earlier to testify that the plaintiff had brought suit against him for defects in the earlier house, many of which were like those now claimed to be found in the home the defendant built, but that the case was settled without trial.

Should the trial court rule that the witness’s offered testimony is admissible?

A Yes, as proper impeachment because the plaintiff will have an opportunity to explain or deny the witness’s statement.

B Yes, because the plaintiff failed to object to the defendant’s questions on cross-examination relative to the prior suit.

C No, because the best evidence of the former suit is the court record.

D No, because its probative value is substantially outweighed by the danger that it will confuse the issues and waste time.

A

The trial court should rule the witness’s testimony inadmissible because its probative value is substantially outweighed by the danger that it will confuse the issues and waste time. Where a witness makes a statement not directly relevant to the issues in the case, the rule against impeachment (other than by cross-examination) on a collateral matter applies to bar the opponent from proving the statement untrue either by extrinsic contradictory facts or by a prior inconsistent statement. The purpose of the rule is to avoid the possibility of unfair surprise, confusion of issues, and undue consumption of time. An issue is considered collateral if it would not be admissible other than to contradict the testimony. Evidence that a person has previously filed similar claims is generally inadmissible to show the invalidity of the present claim. At best, this evidence shows the plaintiff’s tendency toward litigation. Unless there is evidence that the previous claim was false, the probative value of such evidence is deemed outweighed by the risk of confusion of the issues. Because the prior suit would not be the subject of proof independent of impeachment, it is a collateral matter, and extrinsic evidence, such as the witness’s testimony, is inadmissible. (A) is wrong because the testimony is not proper impeachment and is inadmissible. Further, this choice states the foundational requirement for introducing a prior inconsistent statement. This is not a prior inconsistent statement and, if the testimony were admissible, the opportunity to explain or deny would not be required. (B) is wrong because the failure to object merely meant that the plaintiff’s answer to the question was admitted into evidence; it does not change the fact that the matter is collateral. Because it is a collateral matter, extrinsic evidence will not be permitted. (C) is wrong for two reasons: First, the suit is a fact that exists independently of the court record, and thus the best evidence rule would not apply. Further, as stated above, extrinsic evidence of any kind is not admissible on a collateral matter; the defendant is limited to cross-examination for impeachment in these circumstances.

108
Q

An antiques purchaser who did not speak English sued a dealer for breach of contract, alleging that he had agreed to sell her an antique chair for $15,000 but had refused to accept her certified check when she came to pick up the chair. At the trial, the purchaser, through an interpreter, testified that she asked her brother to communicate to the dealer her offer to purchase the chair. She wishes to testify that her brother told her, “The dealer has agreed to sell you the chair for $15,000.” The agreement was not reduced to writing and the brother died a few days after that conversation.

If the jurisdiction has a typical “Dead Man Act,” what effect will the Act have upon the admissibility of the purchaser’s conversation with her brother?

A It will render the conversation inadmissible because a civil action is involved.

B It will render the conversation inadmissible because the purchaser is an interested party.

C None, because the dealer is not a protected party.

D None, because a civil action is involved.

A

The Dead Man Act will have no effect on the admissibility of the purchaser’s conversation with her brother because the dealer is not a protected party.

A typical Dead Man Act provides that a party or person interested in the event, or her predecessor in interest, is incompetent to testify to a personal transaction or communication with a deceased when such testimony is offered against the representative or successor in interest of the deceased. Such statutes are designed to protect those who claim directly under the decedent from perjured claims.

Here, the dealer is not a representative or successor in interest of the brother, such as an executor, administrator, heir, legatee, or devisee. Therefore, the dealer is not a protected party for purposes of a Dead Man Act. Because the testimony of the purchaser is not being offered against a representative or successor in interest of the decedent (her brother), the Dead Man Act is inapplicable.

Regarding (A), it is true that the bar to competency created by a Dead Man Act applies only to civil cases. However, the mere fact that a civil action is involved will not trigger applicability of a Dead Man Act. As explained above, the absence of someone who is deemed to be a protected party will prevent a Dead Man Act from having any effect. Thus, (A) is incorrect.

Regarding (B), it is true that the purchaser is an interested party (i.e., she stands to gain or lose by the direct and immediate operation of a judgment in this case). Nevertheless, (B) is incorrect because a Dead Man Act requires not only an interested person but a protected party. As has been noted, the dealer is not a protected party.

(D) is incorrect because it is based on the assumption that a Dead Man Act does not apply to civil cases. In fact, such statutes apply only to civil cases, and not to criminal cases.

109
Q

A plaintiff sues her employer for sexual harassment. During the trial, the plaintiff attempts to introduce into evidence company records that include written complaints from other employees alleging that they too were sexually harassed by the employer. The defense objects to the admission of the records on the basis of hearsay.

Should the objection be sustained?

A Yes, because the records are hearsay not within any recognized exception.

B No, because the records qualify under the business records exception.

C No, because the records qualify as a statement against interest.

D No, because the records are not hearsay.

A

The court should sustain the objection because the records are hearsay not within any recognized exception.

Hearsay is a statement, other than one made by the declarant while testifying, offered into evidence to prove the truth of the matter asserted.

Here, the records are being offered to prove that the employer sexually harassed other employees, to support the plaintiff’s contention that the employer sexually harassed her. Because the statements are offered to prove the truth of the matter asserted, they are hearsay, and because there is no recognized exception that would allow the records to be admitted, they must be excluded. Therefore, (D) is incorrect.

(B) is incorrect. The business records exception applies to records or writings made in the course of a regularly conducted business activity by one who was under a duty to do so. Here, because the employees were not under a business duty to file their claims, the business records exception does not apply to their statements.

(C) does not agree with the facts. A statement against interest is a hearsay exception allowed when a declarant is unavailable. Here, there is no showing of unavailability, and also the employees said nothing against their interests.

110
Q

state court is LEAST likely to take judicial notice of which of the following?

A The blood type that occurs with greatest frequency in the population is O-positive.

B Main Street, upon which the courthouse is situated, runs north and south.

C The sun rose at 6:52 a.m. on Friday, December 12, of last year.

D In Australian law, there is no private action for environmental issues.

A

The state court is least likely to take judicial notice of the Australian law. Most state courts will not take judicial notice of the law of a foreign country. Note that foreign law is a legislative fact and thus would not be covered by Federal Rule 201, which covers only adjudicative facts, even if the case were in federal court. (A) and (B) are incorrect because they are notorious facts (i.e., facts of common knowledge in the community), and (C) is incorrect because it is a manifest fact (i.e., a fact capable of certain verification by resort to easily accessible sources of unquestionable accuracy). Both manifest and notorious facts are appropriate for judicial notice, and under the Federal Rules, notice must be taken of these facts if so requested by a party.

111
Q

A plaintiff has brought suit in federal district court against the Social Security Administration because it denied her retirement benefits on the asserted ground that she had not reached the requisite age to qualify. At trial, the plaintiff introduced into evidence a family Bible given to her by her father in which is inscribed her date of birth, showing her to be 65 years old. The government introduced a certified copy of the plaintiff’s birth certificate, which shows her age to be 55. The court admitted both items over objection of the nonpropounding party.

Was this error?

A Yes, as to the Bible only, because it contained inadmissible hearsay.

B Yes, as to the birth certificate only, because it was not authenticated by the custodian of records.

C Yes, as to both, for the reasons stated in the previous answers.

D No, both records were admissible.

A

It was not error to introduce either item of evidence, even though both contain hearsay. Hearsay is a statement, other than one made by the declarant while testifying at the trial or hearing, offered in evidence to prove the truth of the matter asserted. Here, both items of evidence are being offered to prove the truth of what they are asserting—the date of the plaintiff’s birth. However, they both fall within exceptions to the general rule that hearsay is not admissible at trial. Under Federal Rule 803(13), statements of fact concerning personal or family history contained in family Bibles, engravings on tombstones, etc., are admissible (regardless of whether the declarant is available). The plaintiff’s Bible is therefore admissible, and (A) and (C) are incorrect. The certified copy of the birth certificate is also admissible hearsay under Federal Rule 803(9), which admits official records of births, deaths, and marriages. (B) is incorrect because official records are self-authenticating when they are certified [Fed. R. Evid. 902]; the custodian need not authenticate them in court.

112
Q

An insured purchased a life insurance policy on his life, naming his brother as beneficiary. Fifteen years ago, the insured traveled overseas on what was supposed to be a six-month trip, but has not been heard from since. The brother contacted the insurance company, which refused to pay the claim on the basis that there was no evidence that the insured was dead. The brother filed suit against the insurance company to collect the proceeds under the policy. The jurisdiction in which the action has commenced has a statute that states that a person is presumed dead if missing from the jurisdiction for seven years, and if no one in the jurisdiction has heard from the person in those seven years.

Assume that no other evidence is admitted at the trial on the issue of the insured’s death. Which of the following is the most accurate statement?

C The judge must rule as a conclusive presumption that the insured is dead.

D The jury must find that the insured is dead.

A

The jury must find that the insured is dead.

Because the basic facts that support the presumption were proven at trial, and no other evidence was introduced, the jury must find in accordance with the presumption, because the other party did not meet its burden of going forward with rebuttal evidence.

(C) is wrong because the presumption regarding the insured’s death is a rebuttable presumption. A rebuttable presumption will have no force or effect when sufficient contrary evidence is introduced. A conclusive presumption is really a rule of substantive law and cannot be rebutted by contrary evidence.

113
Q

Lay Witness:

A

Testimony involving sense recognition (e.g., an object was heavy and bulky), a state of emotion (e.g., a person seemed cheerful), and whether a person was intoxicated are admissible because they are based on the perception of the witness rather than on specialized knowledge.

When agency or authorization is in issue, a lay witness generally may not state a conclusion as to her authorization. Thus, a lay witness cannot testify that her employer directly authorized her to enter into a contract where that is at issue in the case.

114
Q

Expert Witness:

A

Under Federal Rule 703, the expert may base an opinion upon facts not known personally but supplied to him outside the courtroom, and such facts need not be in evidence or even of a type admissible in evidence, as long as the facts are of a kind reasonably relied upon by experts in the particular field. However, if the facts are of a type inadmissible in evidence, the proponent of the expert opinion must not disclose those facts to the jury unless the court determines that their probative value in assisting the jury to evaluate the expert’s opinion substantially outweighs their prejudicial effect.

An expert may give opinion testimony on direct examination without disclosing the basis of the opinion, unless the court orders otherwise. However, the expert may be required to disclose such information on cross-examination.

An expert’s opinion may be based upon the evidence introduced at the trial and communicated to the expert by counsel, usually in the form of a hypothetical question.

115
Q

An expert’s opinion may be based on one or more of the following sources of information:

A
  1. facts that he knows from his own observation,
  2. facts presented in evidence at the trial and submitted to the expert (i.e. can sit in the courtroom and listen in and then testify), or
  3. facts not in evidence that were supplied to the expert out of court and which are of a type reasonably relied upon by experts in the particular field in forming opinions on the subject.
116
Q

Impeachment:

A

Extrinsic evidence of “bad acts” is not permitted, even where the witness denies committing the act on cross-examination. If the witness denies the act, the cross-examiner, acting in good faith, may generally continue the cross-examination after a denial in the hope that the witness will change his answer. The cross-examiner cannot refute the answer by calling other witnesses or producing other evidence.

Under Federal Rule 608(b), subject to the discretion of the trial judge, a witness may be interrogated on cross-examination with respect to any specific act that may impeach his character and show him to be unworthy of belief, as long as the act is probative of truthfulness (i.e., an act of deceit or lying). A conviction of a crime is not necessary under this rule. Cheating on one’s taxes is lying, so this would be a specific act of misconduct reflecting on the brother’s character for truthfulness.

117
Q

An admission (i.e., statement attributable to an opposing party) is frequently not the statement or act of the party against whom the admission is offered at trial. A party can be held vicariously liable for statements made by people with the following relationships to the party:

A
  • Authorized Spokesperson: The statement of a person authorized by a party to speak on its behalf e.g., statement by company’s press agent) can be admitted against the party as an admission.
  • Principal-Agent: Statements by an agent concerning any matter within the scope of her agency or employment, made during the existence of the agency or employment relationship, are admissible against the principal. Therefore, if a truck driver-employee has an accident while on the job and admits that she was negligent, this admission may be introduced against her employer even if she was not authorized to speak for the employer. The statement can be attributed to the employer, provided (i) it was made while the person was employed by the employer (not before or after the period of employment) and (ii) the statement related to the employment.
  • Partners: After a partnership is shown to exist, an admission of one partner, relating to matters within the scope of the partnership business, is binding upon her co-partners since, as to such matters, each partner is deemed the agent of the others.
  • Co-Conspirators: The Supreme Court has held that admissions of one conspirator, made to a third party in furtherance of a conspiracy to commit a crime or a civil wrong, at a time when the declarant was participating in the conspiracy, are admissible against co-conspirators.
  • In contrast, statements of a party are not receivable against her co-plaintiffs or co-defendants merely because they happen to be joined as parties to the action.
118
Q

Impeachment with prior inconsistent statement

A

A testifying witness’s prior inconsistent statement made at a deposition is not hearsay because the statement was made under penalty of perjury. Prior inconsistent statements made under oath at a prior trial or proceeding, or in a deposition, are considered nonhearsay under the Federal Rules as long as the declarant is now testifying and subject to cross-examination (i.e., willingly responding to questions).

Under Federal Rule 806, the credibility of an unavailable declarant may be attacked by evidence that would be admissible if the declarant had testified as a witness. There is no requirement that a declarant must be present at trial to be impeached. If the declarant is impeached with evidence of her prior inconsistent statement, the foundational requirement that she must explain or deny her statement does not apply. Furthermore, where the declarant’s credibility is impeached, it may also be rehabilitated.

Because prior inconsistent statements are generally hearsay, they often are admissible only for purposes of impeachment. The Federal Rules do categorize a testifying witness’s prior inconsistent statement as nonhearsay if it was made under penalty of perjury at a prior trial, hearing, or proceeding, or in a deposition_. Here, of course, the employee’s letter to the friend was not made under oath, so it is hearsay. However, it is still admissible as substantive evidence because it falls within an exception to the hearsay rule. Under Rule 803(3), a statement of a declarant’s then-existing state of mind is admissible as a basis for a circumstantial inference that the declarant acted in accordance with his state of mind. [See also Mutual Life Insurance Co. v. Hillmon (1892)] The employee’s statement that he was going to do electrical work on the home is admissible as circumstantial evidence tending to show that he followed through with his plans and did the electrical work, which is what the statement is being offered to establish. In this case, therefore, the letter should be admissible as both substantive and impeachment evidence_

119
Q

Spousal immunity

A

When the privilege of spousal immunity is invoked, a married person whose spouse is the defendant in a criminal case may not be called as a witness by the prosecution, and a married person may not be compelled to testify against his spouse in any criminal proceeding. Spousal immunity may be invoked in criminal cases only.

The privilege lasts only during the marriage and terminates upon divorce or annulment. However, if a marriage exists, the privilege can be asserted even as to matters that took place before the marriage.

120
Q

Voluminous writings:

A

The original document or best evidence rule generally requires the original writing to be produced when the terms of the writing are sought to be proved and are material to the case. [Fed. R. Evid. 1002] However, under Federal Rule 1006, the contents of voluminous writings that are otherwise admissible may be presented in the form of a chart as long as the original documents are available to the other party for examination and copying.

121
Q

Cross-examination of D: While cross-examining a defendant on trial for robbery and assault with a deadly weapon, the prosecutor asks him whether he was convicted of fraud within the previous year. Proper?

A

The question is proper. The defendant has taken the stand in his own defense, and therefore the prosecutor can attack his credibility as a witness. Under Federal Rule 609, evidence of conviction of a crime requiring proof of an act of dishonesty or false statement can always be used to attack a witness’s character for truthfulness.

122
Q

One of the general requirements for admissibility of real evidence is that

A

be authenticated; i.e., that it be identified as being what its proponent claims it is.

If the evidence is of a type that is likely to be confused or can be easily tampered with, the proponent of the object must present evidence of chain of custody. The proponent must show that the object has been held in a substantially unbroken chain of possession. It is not necessary to negate all possibilities of substitution or tampering; rather, what is required is to show adherence to some system of identification and custody.

i.e.

Here, the proponent of the blood sample (the prosecution) has not shown what the officer did with it after leaving the crime scene. There is no showing that the vial was placed directly in a properly secured area so as to diminish the possibility of tampering. In short, it has not been demonstrated that there was adherence to some defined system of identification and custody. In the absence of a substantially unbroken chain of custody, the evidence is inadmissible for lack of proper authentication

123
Q

Treatises:

A

Under the Federal Rules, learned treatises can be used either for impeachment or as substantive evidence.

Reliability of a publication may be established by: (i) the direct testimony or cross-examination admission of the expert, (ii) the testimony of another expert, or (iii) judicial notice.

The Federal Rules recognize an exception to the hearsay rule for learned treatises and admit them as substantive evidence if:

(i) the expert is on the stand and it is called to his attention, and (ii) it is established as reliable authority

124
Q

A beneficiary has filed a petition in the probate court to contest the validity of a testator’s will. The beneficiary contends that when the testator executed the will eight years before, he had a severe mental illness and was incapable of forming a valid testamentary intent. In support of this contention, the beneficiary seeks to offer an affidavit prepared by the testator’s former attorney, which states that she was asked to prepare a will for the testator just four months before this will was made. The attorney had refused to do so because it was her opinion that the testator seemed incoherent and paranoid.

How should the judge rule on the admissibility of this affidavit?

A

The judge should rule this affidavit to be inadmissible hearsay. This affidavit is clearly hearsay, and there is nothing in the facts that shows that it is admissible under any of the exceptions to this rule. Hence, (A) is wrong.

(B) is wrong because the observations of the attorney would not be deemed a “communication received from the client.” Also, while the attorney-client privilege generally survives the client’s death, it does not apply to communications relevant to an issue between parties who are claiming through the same deceased client, such as in the probate proceedings here.

(D) is incorrect because a lay person could probably testify to her opinion in this situation since it is rationally based on her own perception, it is helpful to a determination of a fact in issue, and it is not based on scientific, technical, or other specialized knowledge.

125
Q

A plaintiff sued a defendant for damages suffered when a load of bricks fell off the defendant’s truck directly in front of the plaintiff while she was driving on a highway. The plaintiff charged that the defendant was negligent in supplying his truck with a defective load chain clamp, which helped tie the load to the bed of the truck, and in failing to secure the load properly on the truck. The plaintiff calls a witness who testifies that he was formerly employed as a truck driver and is an acquaintance of the defendant. The witness further testifies that immediately prior to the accident he had coffee with the defendant at a cafe, and mentioned to the defendant that the tie chains holding the load of bricks looked kind of loose.

Assuming proper objection by the defendant’s attorney, how should the court rule on the admissibility of such testimony?

A

The witness’s testimony is admissible nonhearsay. The statement by the witness is not being offered to prove the truth of the matter asserted therein and thus is not hearsay. Hearsay is a statement made out of court by the declarant, offered in evidence to prove the truth of the matter asserted. [Fed. R. Evid. 801(c)] Although hearsay is inadmissible (unless an exception to the hearsay rule is applicable), a statement that would be inadmissible hearsay to prove the truth thereof may be admitted to show the statement’s effect on the listener or reader. Thus, in a negligence case, where knowledge of a danger is at issue, a person’s warning statement is admissible for the limited purpose of showing knowledge or notice on the part of a listener. Here, one of the theories of recovery underlying the plaintiff’s lawsuit is that the defendant negligently failed to secure the load. Therefore, the plaintiff must show that the defendant either knew or should have known that the load was not properly secured. Consequently, the witness’s statement that the chains looked loose is admissible to show that the defendant had notice of the possible danger. If this same out-of-court statement were offered to show that its contents were true (i.e., that the chains were in fact loose), then it would constitute hearsay, but because the statement is offered to show notice to the defendant of a possible danger, it is nonhearsay

126
Q

A woman was struck by a brick with her name scrawled on it that was thrown through her bedroom window. The victim believes that her ex-boyfriend, who is a gang member, threw the brick because she has become active in anti-gang groups, but she did not actually see him throw it.

If the ex-boyfriend is arrested and put on trial for battery, which of the following items of the victim’s proposed testimony is LEAST likely to be admitted?

A The victim recently moved to a new apartment and only her ex-boyfriend and a few family members knew its location.

B The victim had testified against a member of her ex-boyfriend’s gang last month in a drug case.

C On another occasion, the victim had seen her ex-boyfriend throw a rock through the window of a rival street gang member.

D Immediately after the brick went through her window, the victim heard a voice she recognized as her ex-boyfriend’s yell, “If you don’t start minding your own business, you’ll get a lot worse than this next time!”

A

Evidence of the defendant’s other crimes or misconduct is admissible only if relevant to some issue other than the defendant’s character or propensity to commit the crime charged. Such acts would be admissible to show motive, intent, absence of mistake, identity, or a common plan or scheme. Of these, the only one possibly relevant to these facts is identity. Evidence that the accused committed prior criminal acts that are so distinctive as to operate as a “signature” may be introduced to prove that the accused committed the act in question. Merely throwing an object, such as a brick, through a window could not be considered so distinctive as to operate as a signature. Thus, this evidence would not show identity. The only possible reason for offering the evidence is to show the ex-boyfriend’s propensity to commit the crime charged, in which case the testimony will be inadmissible.

127
Q
A